You are on page 1of 96

Chapter 5

Continuity and Differentiability

Solutions

SECTION - A
Objective Type Questions (One option is correct)

Limit, Continuity and Differentiability

x
1. If the function f ( x )  (1  x ) tan is continuous at x = 1, then f(1) =
2
2 
(1) (2) (3) 0 (4) 2
 2
Sol. Answer (1)
⎛ x ⎞
f(1) = lim f ( x ) = lim (1  x ) tan ⎜ ⎟
x 1 x 1 ⎝ 2 ⎠
⎛ ⎞
= hlim (1  (1  h )) tan ⎜ (1  h) ⎟
0 ⎝ 2 ⎠
⎛  h ⎞
= hlim ( h)tan ⎜  ⎟
0 ⎝2 2 ⎠
⎛ h ⎞
= hlim ( h) ⎜  cot
0 ⎝ 2 ⎟⎠

.h
2 2 2
= lim  
h 0 ⎛ h ⎞  
tan ⎜ ⎟
⎝ 2 ⎠

⎧ ⎛ 1⎞
⎪ x sin ⎜ ⎟ , x  0
2. Let f ( x )  ⎨ ⎝x⎠
⎪k , x 0

then f(x) is continuous at x = 0, if
(1) k = 1 (2) k = 0 (3) k = 2 (4) k = –1

Aakash Educational Services Pvt. Ltd. Regd. Office : Aakash Tower, 8, Pusa Road, New Delhi-110005 Ph.011-47623456
136 Continuity and Differentiability Solutions of Assignment (Set-2)

Sol. Answer (2)

1
lim x sin 0
x 0 x
 k = 0 will make f continuous at 0.

1
(∵ lim ( x )  0 and sin ⎛⎜ ⎞⎟  1 for x  0 )
x 0 ⎝x⎠

⎧ 3 | x |  4 tan x
⎪ , x0
3. Let f ( x )  ⎨ x
⎪⎩ k , x 0

then f(x) is continuous at x = 0 for


(1) k = 7 (2) k = 1 (3) No values of k (4) k = 2
Sol. Answer (3)

3 | x |  4 tan x
lim f ( x ) = lim
x  0 x 0 x

3(  x )  4 tan x
= lim
x 0 x

3 x 4 tan x
= lim  lim
x 0 x x 0  x

= –3 + 4 = 1

3 | x | 4 tan x
and lim f ( x ) = lim
x  0 x 0 x

⎛ 3 x 4 tan x ⎞
= lim ⎜ x  x ⎟
x 0 ⎝ ⎠
=3+4=7

lim f ( x )  lim f ( x )
x 0  x 0 

 f(x) is not continuous at x = 0, whatever k may be.

⎧⎪ x  a , x  1
4. Let f ( x )  ⎨ 2 then f(x) is continuous at x = 1 for
⎪⎩ax  1 , x  1

(1) a = 0 (2) a=1 (3) All a  R (4) No value of a


Sol. Answer (3)
f(1) = a + 1

lim f ( x )  lim ( x  a )  (1  a )
x  1 x 1

Aakash Educational Services Pvt. Ltd. Regd. Office : Aakash Tower, 8, Pusa Road, New Delhi-110005 Ph.011-47623456
Solutions of Assignment (Set-2) Continuity and Differentiability 137

2
and lim f ( x )  lim (ax  1)  a  1
x 1 x 1

 f(x) is continuous at x = 1, whatever ‘a’ may be.

x (e sin x  1)
5. If the function f ( x )  is continuous at x = 0, then f(0) =
1  cos x

1
(1) 1 (2) 0 (3) 2 (4)
2
Sol. Answer (3)

x (e sin x  1)
f(0) = lim f ( x )  lim
x 0 x  0 1  cos x

e sin x  1 sin x x2
= lim   =2
x  0 sin x x 1  cos x

x (2 x  1)
6. Let f ( x )  for x  0. What choice of f(0), if any, will make f(x) continuous at x = 0?
1  cos x

1 1
(1) log 2 (2) log2 (3) (4) 2 log 2
2 2log2

Sol. Answer (4)


x
lim f ( x ) = lim x (2  1)
x 0 x  0 1  cos x

2x  1 x2
= lim 
x 0 x 1  cos x

⎛ ⎞
⎜ 1⎟
= (log2)  ⎜ ⎟  2log2
⎜⎜ 1 ⎟⎟
⎝2⎠

⎧[ x ]  [  x ] , x  0
7. If f ( x )  ⎨ , where [.] denotes the greatest integer function, then f(x) is continuous at x = 0,
⎩  , x 0
for  =
(1) –1 (2) 0 (3) 1 (4) No value is possible
Sol. Answer (1)

lim f ( x ) = lim [ x ]  [ x ]
x  0 x 0

= lim [ x ]  lim [ x ]
x 0 x 0

= –1 + 0 = –1

Aakash Educational Services Pvt. Ltd. Regd. Office : Aakash Tower, 8, Pusa Road, New Delhi-110005 Ph.011-47623456
138 Continuity and Differentiability Solutions of Assignment (Set-2)

lim f ( x ) = lim [ x ]  [  x ]
x  0 x  0

= lim [ x ]  lim [  x ]
x  0 x  0

= 0 + (–1) = –1
  = –1 will make f continuous at x = 0.

⎧ sin[ x ]
⎪ , [x]  0
8. Let f ( x )  ⎨ [ x ] , then f(x) is ([.] denotes the greatest integer function)
⎪ 0 , [x]  0

(1) Continuous at x = 0 (2) Left continuous at x = 0


(3) Differentiable at x = 0 (4) Right continuous at x = 0
Sol. Answer (4)

lim f ( x ) = lim 0  0  f (0)


x  0 x 0

sin[ x ]
lim f ( x ) = lim
x  0 x  0 [x]

sin( 1)
= lim 1
x 0

= sin1  f (0)

1
9. Let f ( x )  sin , x  0. Then f(x) can be continuous at x = 0
x
(1) If f(0) = 1 (2) If f(0) = 0 (3) If f(0) = –1 (4) For no value of f(0)
Sol. Answer (4)
For continuity at x = 0,

1 1
lim sin  lim sin  f (0)
h0 0  h h0 0h

1
But lim sin  lim (a number between 1 and –1)
h0 h h0
 a definite number
 f(0) cannot be a definite number.

⎧⎪ px 2  q , x  [0, 1)
10. If f ( x )  ⎨
⎪⎩ x  1 , x  (1, 2]

and f(1) = 2, then the value of the pair (p, q) for which f(x) cannot be continuous at x = 1 is
(1) (2, 0) (2) (1, –1) (3) (4, 2) (4) (1, 1)

Aakash Educational Services Pvt. Ltd. Regd. Office : Aakash Tower, 8, Pusa Road, New Delhi-110005 Ph.011-47623456
Solutions of Assignment (Set-2) Continuity and Differentiability 139

Sol. Answer (4)


f(x) is continuous at x = 1, if

lim f (1  h )  lim f (1  h )  f (1)  2


h 0 h 0

lim f (1  h )  lim {1  h  1}  2
h0 h0

lim f (1  h)  lim { p(1  h)2  q }  p  q


h 0 h 0

 pq  2
 (1, 1) cannot be the solution.

x f (a )  af ( x )
11. If f(x) is derivable at x = a, then lim
x a x a

(1) f (a ) (2) af (a )  f (a ) (3) f (a )  af (a ) (4) af (a )


Sol. Answer (3)

xf (a )  af ( x )
lim
x a x a

(a  h ) f (a )  af (a  h )
= lim
h0 h

hf (a ) a[f (a )  f (a  h )]
= lim  lim
h0 h h  0 h

f (a  h )  f (a )
= f (a )  a lim  f (a )  af (a )
h 0 h
12. Let f(x) = x|x|, then f(0) is equal to
(1) 1 (2) –1 (3) 0 (4) 1
Sol. Answer (3)

⎧⎪ x 2 ; x  0
f(x) = x |x| = ⎨ 2
⎪⎩ x ; x  0

f (0  h )  f (0) ⎛ h 2  0 2 ⎞
Lf (0)  lim = h  0 ⎜ h ⎟⎟  0
lim ⎜
h0 h ⎝ ⎠

f (0  h )  f (0) h 2  02
Rf (0)  lim = lim 0
h0 h h 0 h

 Lf (0)  Rf (0)

 f (0)  0

Aakash Educational Services Pvt. Ltd. Regd. Office : Aakash Tower, 8, Pusa Road, New Delhi-110005 Ph.011-47623456
140 Continuity and Differentiability Solutions of Assignment (Set-2)

⎧⎪ x  a , x  1
13. Let f ( x )  ⎨ 2 then f(x) is derivable at x = 1, if
⎪⎩ax  1 , x  1

1
(1) a = 1 (2) a=0 (3) a=2 (4) a
2

Sol. Answer (4)


f(1) = a + 1

f (1  h )  f (1) a (1  h )2  1  (1  a )
Lf (1)  lim = lim = 2a
h0 h h0 h

f (1  h )  f (1) 1  h  a  (1  a )
and Rf (1)  lim = lim 1
h0 h h 0 h

1
 f(x) is derivable at x = 1, only if 2a = 1 i.e. if a  .
2

f ( x )  f (1)
14. If f ( x )   25  x 2 , then lim is equal to
x 1 x 1

1 1 1
(1) (2) (3)  24 (4)
24 5 24

Sol. Answer (4)

f ( x )  f (1) 1 ( 2)  1 1
lim  f (1)  
x 1 x 1 2 25  12 24

1
2
15. If f ( x )  ex , x  0 and f(0) = 0 then f(0) is

(1) Not defined (2) 1 (3) e (4) 2


Sol. Answer (1)

1 1 1
1  h 0
(0  h )2  h h2 lim  0
e 0 1 h2 = lim 1 = lim
Rf (0)  lim  lim e h0 h0 1 = h 0 1 
h 0 h h 0 h 2 2 ⎛ 2 ⎞ 2e h
2
eh eh ⎜ 3 ⎟
⎝ h ⎠

1 1 1
 1 
h2
2
e (0  h )
0 ⎛ 1⎞  2 h  lim h 0
Lf (0)  lim  lim ⎜  ⎟ e h = hlim 1 1 = lim  0
h 0 h h 0 ⎝ h ⎠
0
2
h0
2 ⎛ 2 ⎞ h 0 1 
eh eh ⎜  3 ⎟ 2e h
2

⎝ h ⎠

Aakash Educational Services Pvt. Ltd. Regd. Office : Aakash Tower, 8, Pusa Road, New Delhi-110005 Ph.011-47623456
Solutions of Assignment (Set-2) Continuity and Differentiability 141

16. Let the function f(x) be defined as

⎧ ln x  1
⎪ , xe
f (x)  ⎨ x  e
⎪⎩ k , xe

The value of k, for which the function is continuous at x = e, is equal to

1
(1) e (2) (3) e2 (4) –e
e

Sol. Answer (2)

ln x  1 ln x  ln e ln(e  h )  ln e 1
lim  log = log = f (e ) 
x e x  e x e h
x e h 0 e

1
 k .
e

f (x)  3
17. If f be a function of ‘x’ such that f(9) = 9, f(9) = 3, then lim is
x 9 x 3

1
(1) 9 (2) 3 (3) 1 (4)
3

Sol. Answer (2)

f (x)  3 f ( x )  32 x 3
lim = lim 
x 9 x 3 x 9 f ( x )  3 x  32

f (x)  9 x 3
= lim 
x 9 x 9 f (x)  3

f ( x )  f (9) x 3
= lim  lim
x 9 x 9 x 9 f (x)  3

9 3 ⎛ 33 ⎞
= f (9) = 3⎜ ⎟ =3
f (9)  3 ⎝ 9 3⎠

⎧ 1
⎪ , x0
18. If f ( x )  ⎨1  e1/ x then f(x) is
⎪ 0 , x0

(1) Continuous at x = 0 (2) Continuous and differentiable at x = 0


(3) Continuous but not differentiable at x = 0 (4) Discontinuous at x = 0

Aakash Educational Services Pvt. Ltd. Regd. Office : Aakash Tower, 8, Pusa Road, New Delhi-110005 Ph.011-47623456
142 Continuity and Differentiability Solutions of Assignment (Set-2)

Sol. Answer (4)


f(0) = 0

1

1 1 e x 0
lim f ( x )  lim  lim  lim  0
x 0 
x 0  1
x 0 1  ⎛ 1 ⎞ x 0 1 0 1
1 e x ex ⎜e x  1⎟ e x 1
⎜ ⎟
⎝ ⎠

1 1
and lim f ( x )  lim  1
x 0 x 0
1 1 0
1 e x

Thus, lim f ( x )  lim f ( x )


x  0 x  0

 f(x) is discontinuous at x = 0.

19. Let f ( x )  1  1  x 2 , then f(x) is

(1) Continuous on [–1, 1] and differentiable on (–1, 1)


(2) Continuous on [–1, 1] and differentiable on (–1, 0) (0, 1)
(3) Continuous and differentiable on (–1, 1)
(4) Discontinuous on [–1, 1]
Sol. Answer (2)
f(x) is continuous in [–1, 1]

1 1 1 1
But f'(x) =  ( 2 x )
2 2 1 x2
1 1 x2

x
=
2 1 x2 1 1 x2

which does not exist when 1  x 2 = 0, 1

 f(x) is differentiable at all x  [–1, 1] except when x2 = 0, 1.


 f(x) is derivable in (–1, 0)  (0, 1).
20. Domain of differentiability of the function f(x) = |x – 2| cos x is
(1) R (2) R – {2} (3) (0, ) (4) {2}
Sol. Answer (2)
As |x – 2| is not derivable when x – 2 = 0, i.e. when x = 2, therefore f(x) = |x – 2| cos x is derivable at all

(cos x )( x  2)
x  R except at x = 2 and f'(x) = |x – 2| (–sin x) + .
| x 2|

Aakash Educational Services Pvt. Ltd. Regd. Office : Aakash Tower, 8, Pusa Road, New Delhi-110005 Ph.011-47623456
Solutions of Assignment (Set-2) Continuity and Differentiability 143

sin(  [ x  ])
21. Let f ( x )  , where [ ] denotes the greatest integer function, then f(x) is
1  [ x ]2

(1) Continuous and differentiable at all x R (2) Continuous but not differentiable at some x
(3) Differentiable but not continuous at x = 0 (4) Neither continuous nor differentiable at x = 0
Sol. Answer (1)
As [x + ] is always an integer and sine of an integral multiple of  is zero, therefore, numerator of f(x) is 0
for all x  R and denominator is always positive
 f(x) = 0 for all x  R
So, f(x) is a constant function with Df = R
Hence, f(x) is continuous and derivable at all x  R.

2 x2 x2
22. If f ( x )  x    ..... upto . Then
1 x2 (1  x 2 )2

(1) xlim
0
f ( x ) does not exist (2) f(x) is continuous but not differentiable at x = 0

(3) f(x) is discontinuous at x = 0 (4) f(x) is differentiable at x = 0


Sol. Answer (3)
f(0) = 0 + 0 + 0 + ..... = 0

⎪⎧ x2 x2 ⎪⎫
lim f ( x ) = lim ⎨ x 2    .....⎬
2 2 2
x 0 x 0 ⎪
⎩ (1  x ) (1  x ) ⎪⎭

⎪⎧ 1 1 ⎪⎫
= lim x 2 ⎨1  2
 2 2
 .....⎬
x 0
⎩⎪ (1  x ) (1  x ) ⎭⎪

⎧ ⎫ ⎛ 1 ⎞
⎪ 1 ⎪⎪ ⎜∵ x  0,  x  0 ⇒ 0  1 ⎟
2⎪
2
= lim x ⎨ ⎜ 1  x ⎟

x 0
⎪1  1 ⎪ ⎜ ⇒ Series inside the brackets is an infinite G.P. with C.R.  1) ⎟
⎝ ⎠
⎪⎩ 1  x 2 ⎪⎭

2
= lim (1  x )  1
x 0

So, f (0)  lim f ( x )


x 0

⎧ | x  1|
⎪ , x  1
23. Let f ( x )  ⎨ tan1( x  1) . Then f(x) is
⎪1 , x  1

(1) Continuous at x = –1 (2) Differentiable at x = –1


(3) Discontinuous at x = –1 (4) Continuous but not derivable at x = –1

Aakash Educational Services Pvt. Ltd. Regd. Office : Aakash Tower, 8, Pusa Road, New Delhi-110005 Ph.011-47623456
144 Continuity and Differentiability Solutions of Assignment (Set-2)

Sol. Answer (3)

lim f ( x ) = | x  1|
lim

x  1 x  1 
tan1 | x  1|

x 1 tan t S
= lim = 1 (∵ x  –1+  x + 1  0+ and lim  1  lim )
x  1 tan1( x  1) t 0 t S 0 tan1 S

| x  1|
and lim f ( x ) = lim
x  1 x  1 tan1( x  1)

( x  1)
= lim = – 1. Hence f(x) is not continuous at x = –1.
x  1 
tan1( x  1)

f ( x  h)  f ( x  h)
24. The value of lim is equal to
h 0 h
(1) f(x) (2) 0 (3) 2f(x) (4) –f(x)
Sol. Answer (3)

f ( x  h)  f ( x  h) f ( x  h)  f ( x ) f ( x  h)  f ( x )
lim = lim  lim = f'(x) + f'(x) = 2f'(x).
h 0 h h 0 h h0 h

⎛ 1 ⎞⎛ 1 ⎞
25. The value of lim ⎜ 1  x ⎟⎜ ⎟ is
x 0 ⎝ 2 ⎠ ⎝ tan x  4  2 ⎠

(1) loga16 (2) Cannot exist (3) 3n2 (4) 4n2


Sol. Answer (4)

⎛ 2 x  1 ⎞ ⎛ tan x  4  2 ⎞
lim ⎜ ⎟⎜ ⎟
x 0 ⎜ 2 x ⎟⎜ tan x  4  4 ⎟
⎝ ⎠⎝ ⎠

⎛ 2 x  1 ⎞ ( tan x  4  2) 1
lim ⎜ ⎟ 
= x 0 ⎜⎝ x ⎟⎠ tan x 2x
x

⎛4⎞ 1
lim log2 ⎜ ⎟ 
= x 0 ⎝ 1⎠ 2
= 4log2

sin6 x sin x log x


26. If lim  a, lim  b, lim  c then value of a + b + c is
x 0 tan3 x x  x x  x
(1) 1 (2) 2 (3) 3 (4) 4
Sol. Answer (2)

⎛ sin 6 x ⎞
sin 6 x ⎜ ⎟ 6x
lim  lim ⎝ 6x ⎠ 2
a=
x 0 tan3 x x 0 ⎛ tan3 x ⎞
⎜ ⎟ 3x
⎝ 3x ⎠

Aakash Educational Services Pvt. Ltd. Regd. Office : Aakash Tower, 8, Pusa Road, New Delhi-110005 Ph.011-47623456
Solutions of Assignment (Set-2) Continuity and Differentiability 145

sin x
b = lim 0
x  x

⎛ 1⎞
log x ⎜ ⎟ 1
x
c = lim  lim ⎝ ⎠  lim 0 (By L’ Hospital Rule)
x  x x  1 x  

 a+b+c=2+0+0=2

27. Let a  lim x cot x and b  lim x log x , then


x 0 x 0

(1) a = b (2) b>a (3) a=b+1 (4) b=a+1


Sol. Answer (3)
x
a = lim x cot x  lim 1
x 0 x 0 tan x
⎛ 1⎞
log x ⎜ ⎟
b = lim x log x  0      = lim
x
 lim ⎝ ⎠  lim   x   0 (By L’ Hospital Rule)
x 0 x 0 ⎛ 1 ⎞ x 0 1 x 0
⎜ ⎟  2
⎝x⎠ x
 a=b+1
x tan 2 x  2 x tan x
28. lim equals
x0 (1  cos 2 x )2

1 1 1
(1) 1 (2) (3) (4)
3 4 2
Sol. Answer (4)

2 tan x
x  2 x  tan x
x tan2 x  2 x tan x 1  tan2 x
lim = lim
x 0 (1  cos 2 x )2 x 0 4 sin4 x

⎛ 1 ⎞
x tan x ⎜ 2
 1⎟
⎝ 1  tan x ⎠
= lim
x 0 2 sin4 x

⎛ tan x ⎞ ⎛ 1  (1  tan2 x ) ⎞
x ⎜ ⎟x ⎜ ⎟
⎝ x ⎠ ⎝ 1  tan2 x ⎠
= lim
x 0 2 sin4 x

⎛ tan2 x ⎞
x2 ⎜ ⎟
= lim ⎝ 1  tan2 x ⎠
x 0 2 sin4 x
2
⎛ tan x ⎞ 1
x2 ⎜ 2
⎟ x 
⎝ x ⎠ (1  tan2 x ) 1
= lim 4
=
x 0
⎛ sin x ⎞ 4
2
2⎜ ⎟ x
⎝ x ⎠

Aakash Educational Services Pvt. Ltd. Regd. Office : Aakash Tower, 8, Pusa Road, New Delhi-110005 Ph.011-47623456
146 Continuity and Differentiability Solutions of Assignment (Set-2)

(1  x  x 2 )
29. lim is equal to
x  (ln x )3

(1) 2 (2) e2 (3) e–2 (4) Not defined


Sol. Answer (4)

1 x  x2
lim
x  (nx )3

1  2x
= lim 2
, (By L’ Hospital’s rule)
x  ( n x )
3
x

x  2x 2 1 4x
= lim = lim , (By L’ Hospital’s rule)
x  3( nx )2 x  6nx
x

x  4x2 1 8x 1 8x2
= lim = lim , (By L’ Hospital’s rule) = lim =
x  6nx x  6 / x x  6

sin x n
30. m, n I+, then lim equals
x 0 (sin x )m

n
(1) 1, if n < m (2) 0, if n = m (3) (4) 0, if n > m
m
Sol. Answer (4)

m
sin x n n m sin x n ⎛ x ⎞
lim = lim x ⎜ ⎟
x 0 (sin x )m x 0 xn ⎝ sin x ⎠

⎧ 0, n  m
n m ⎪
lim x  ⎨ 1, n  m
x 0 ⎪, n  m

xf (2) – 2f ( x )
31. Let f(2) = 4, f (2) = 4. Then Lt is
x 2 x–2

1
(1) – (2) –2 (3) –4 (4) 3
3
Sol. Answer (3)

x f (2)  2f ( x )
lim
x 2 x 2
Applying L’ Hospital Rule

Aakash Educational Services Pvt. Ltd. Regd. Office : Aakash Tower, 8, Pusa Road, New Delhi-110005 Ph.011-47623456
Solutions of Assignment (Set-2) Continuity and Differentiability 147

= lim f (2)  2f '( x )


x 2

= f (2)  2f (2)
=4–2×4
= –4

2x – 1
32. lim 
x 0
1 x – 1

loge 2
(1) 2 (2) loge2 (3) (4) 2 loge2
2
Sol. Answer (4)
We have,

2x  1
lim
x 0 1 x  1

2x  1
= lim [ 1  x  1]
x 0 x

= 2 ln 2

1– cos(ax 2  bx  c )
33. Let  and  be the distinct root of ax2 + bx + c = 0, then lim is equal to
x  ( x –  )2

1 a2 a2
(1) (  – ) 2 (2) – (  – ) 2 (3) 0 (4) (  – ) 2
2 2 2
Sol. Answer (4)
,  are the roots of ax2 + bx + c = 0
Hence ax2 + bx + c = a(x – ) (x – )

1  cos(ax 2  bx  c ) 1  cos(a( x   )( x  ))


lim = xlim
x  2
( x  )  ( x   )2

⎛a ⎞
2 2sin2 ⎜ ( x   )( x  ) ⎟
⎛a⎞ 2 ⎝ 2 ⎠
= lim ⎜ ⎟ ( x  ) 2
x  ⎝ 2 ⎠ ⎛a⎞ 2 2
⎜ 2 ⎟ ( x   ) ( x  )
⎝ ⎠

a2
= × 2 (– )2 × 1
4

a2
= ( – )2
2

Aakash Educational Services Pvt. Ltd. Regd. Office : Aakash Tower, 8, Pusa Road, New Delhi-110005 Ph.011-47623456
148 Continuity and Differentiability Solutions of Assignment (Set-2)

sin(  cos2 x )
34. The value of lim equals
x 0 x2


(1) – (2)  (3) (4) e
2
Sol. Answer (2)

sin( (1  sin2 x ) sin(  sin2 x )  sin2 x


= × 
x2  sin2 x x2

35. lim
x   x  x  x equals 
1
(1) (2) 2 (3) 3 (4) 4
2
Sol. Answer (1)

lim
x 
 x x  x 

= lim
 x x  x  x x  x 
x 
 x x  x 
x x x
= lim
x 
x x  x

x
= lim
x  1

x 1 x 2  x

1
= lim 1

1 x 
x 
1 2

2 1

1
=
2

⎛ 1 1 ⎞
36. lim ⎜ 2  ⎟ equals
x 0 ⎝ x tan2 x ⎠

1 2 1
(1) 0 (2) (3) (4)
3 3 2

Aakash Educational Services Pvt. Ltd. Regd. Office : Aakash Tower, 8, Pusa Road, New Delhi-110005 Ph.011-47623456
Solutions of Assignment (Set-2) Continuity and Differentiability 149

Sol. Answer (3)

1 1 (tan x )2  x 2
lim  = xlim
x 0 x2 (tan x )2 0 ( x tan x )2

(tan x  x )(tan x  x )
= lim 2
x 0
⎛ tan x ⎞
⎜x x⎟
⎝ x ⎠

(tan x  x )(tan x  x )
= xlim
0 ( x 2 )2

⎛ x3 2 5 ⎞⎛ x3 2 5 ⎞
⎜x   x  ....  x ⎟⎜ x   x  ....  x ⎟
⎝ 3 15 ⎠ ⎜⎝ 3 15 ⎟

= lim
x 0 x4

2
=
3

1
⎛ 4x  9x ⎞x
37. lim ⎜ ⎟ equals
x  0⎜ 2 ⎟
⎝ ⎠

(1) 2 (2) 6 (3) 16 (4) 112


Sol. Answer (2)

1/ x
⎛ 4x  9x ⎞
lim ⎜ ⎟
x0 ⎜ 2 ⎟
⎝ ⎠

⎛ 4 x 9 x ⎞ 1 ⎛ 4 x 9 x 2 ⎞ 1 ⎛ 4 x 1 9 x 1 ⎞ 1
lim ⎜ 1⎟. lim ⎜ ⎟. lim ⎜  ⎟.
x 0⎜⎝ 2 ⎟x
⎠ x 0⎜⎝ 2 ⎟x
⎠ x 0⎜⎝ 2 2 ⎟⎠ x
= e e e

1 4 x 1 1 9 x 1 1 1
lim  lim ( n4  n9) (2n2 2n3)
= e 2 x 0 x 2 x 0 x  e 2  e 2

n2 n3
= e .e =2×3=6

38. If lim  cos x  a sin bx 


1/ x
 e 2 , then the values of a and b can be
x 0

1
(1) a = 1, b = –2 (2) a  2 2, b  2 (3) a  2 2, b  (4) a = –2, b = 1
2
Sol. Answer (3)

lim (cos x  a sin bx )1/ x  e 2


x 0

Aakash Educational Services Pvt. Ltd. Regd. Office : Aakash Tower, 8, Pusa Road, New Delhi-110005 Ph.011-47623456
150 Continuity and Differentiability Solutions of Assignment (Set-2)

a sin bx
lim
 e x 0 x  e2
 eab = e2
 ab = 2.

1
Clearly a = 2 2 and b = satisfy ab = 2
2

⎛ [ x ]3 ⎡ x ⎤ 3 ⎞
39. lim ⎜ – ⎢ ⎥ ⎟ is, (where [ ] is G.I.F.)
x 2 ⎜ 3 ⎣ 3 ⎦ ⎟⎠

64 8
(1) 0 (2) (3) (4) None of these
27 3
Sol. Answer (3)

⎛ [ x ]3 ⎡ x ⎤ 3 ⎞ 8
lim ⎜  ⎢ ⎥ ⎟ = ⎛⎜  0 ⎞⎟ = 8
⎜ 3 ⎣3⎦ ⎠ ⎟
x 2 ⎝ ⎝3 ⎠ 3

( 4 x – 1)3
40. The value of f(0) so that f ( x )  is continuous everywhere is
x ⎛
⎜ x 2 ⎞⎟
sin · log⎜1 
4 ⎝ 3 ⎟⎠

(1) 3(log 4)3 (2) 4(log 4)3 (3) 12(log 4)3 (4) 15(log 4)3
Sol. Answer (3)

1
1
⎛ x2 ⎞ x2
x x log ⎜ 1  ⎟. 3
sin . ⎜ 3 ⎟⎠ 3 ⎛ 4x  1⎞ x3
(4 x  1)3 x 3 4 4 ⎝ 1 1
lim f ( x ) = lim . . = ⎜ ⎟   
x 0 x 0 x3 x x2 ⎜ x ⎟ ⎛ ⎞ x x2 x4 x6
⎝ ⎠
4 ⎜ sin x / 4 ⎟ 4 3  9  27  ...
3 ⎜ ⎟
⎜⎜ x ⎟⎟
⎝ 4 ⎠
 f(0) = 12(log4)3

⎧ x 2  3x  p
⎪⎪ 2
,x 1
41. If f ( x )  ⎨ 2( x – 1) is continuous at x = 1 then
⎪ 5
, x 1
⎩⎪ 4

(1) p = 2 (2) p=0 (3) p = –4 (4) None of these


Sol. Answer (3)

x2  3x  p 5
lim 
2 4
x 1 2( x  1)

Aakash Educational Services Pvt. Ltd. Regd. Office : Aakash Tower, 8, Pusa Road, New Delhi-110005 Ph.011-47623456
Solutions of Assignment (Set-2) Continuity and Differentiability 151

x 2  3x  p 5
 lim 
x 1 2( x  1)( x  1) 4

For the existence of limit (x – 1) should be a factor of x2 + 3x + p


Hence 1 + 3 + p = 0  p = –4

1
42. The number of points at which the function f ( x )  is discontinuous is
log | x |

(1) 1 (2) 2 (3) 3 (4) 4


Sol. Answer (3)

1
f (x) 
log | x |

Clearly f(x) is not defined at x = 0, –1, 1


Hence the given function is not continuous at 3 points (x = 0, –1, 1)

⎧ – ⎛⎜ 1  1 ⎞⎟
⎜ | x| x ⎟
⎪ ⎝ ⎠
, x  0 , then f(x) is
43. If f ( x )  ⎨ xe

⎩0 ,x  0

(1) Continuous for all x but not differentiable (2) Neither differentiable nor continuous
(3) Discontinuous everywhere (4) Continuous as well as differentiable for all x
Sol. Answer (1)

⎧ ⎛ 1  1 ⎞
⎪ ⎜ ⎟
f ( x )  ⎨ xe ⎝ |x| x ⎠ , x  0
⎪ 0, x 0

⎧ x, x0
⎪⎪ 2/ x
f ( x )  ⎨ xe , x 0
⎪ 0, x 0
⎪⎩
LHL = 0
RHL = 0
f (0) = 0
 The function is continuous.
h
LHD = lim 1
h 0 h

he 2/ h
RHD = lim 0
h 0 h
LHD  RHD
Hence f(x) is not differentiable.

Aakash Educational Services Pvt. Ltd. Regd. Office : Aakash Tower, 8, Pusa Road, New Delhi-110005 Ph.011-47623456
152 Continuity and Differentiability Solutions of Assignment (Set-2)

x
44. The set of points where f ( x )  is differentiable is
1 | x |

(1) (–, –1)  (–1, ) (2) (–, ) (3) (0, ) (4) (–, 0)  (0, )
Sol. Answer (2)

x
f(x) = 1 | x |

⎧ x
⎪⎪1  x , x  0
 f(x) = ⎨
⎪ x , x0
⎪⎩1  x

⎧ 1
⎪ , x0
2
⎪ (1  x )
 f'(x) = ⎨
⎪ 1 , x 0
⎪ (1  x )2

Here we observe that


LHD = 1 = Lf'(0)
RHD = 1 = Rf'(0)
 Function is differentiable over (–,)
45. The function |x2 – 3x + 2| + cos |x| is not differentiable at x =
(1) –1, 2 (2) –1, –2 (3) 1, 2 (4) –2, 1
Sol. Answer (3)

⎧ x 2  3 x  2  cos x, x0

2
⎪⎪ x  3 x  2  cos x, 0  x  1
f (x)  ⎨
⎪  x 2  3 x  2  cos x, 1  x  2
⎪ 2
⎪⎩ x  3 x  2  cos x, x2

⎧ 2 x  3  sin x, x0
⎪ 2 x  3  sin x 0  x  1

f '( x )  ⎨
⎪ 2 x  3  sin x, 1  x  2
⎪⎩ 2 x  3  sin x, x2

At x = 0
LHD = –3
RHD = –3 (x) is differentiable at x = 0

Aakash Educational Services Pvt. Ltd. Regd. Office : Aakash Tower, 8, Pusa Road, New Delhi-110005 Ph.011-47623456
Solutions of Assignment (Set-2) Continuity and Differentiability 153

At x = 1
LHD = –1 – sin1
RHD = 1 – sin1  f(x) is not differentiable at x = 1
At x = 2
LHD = –1 – sin2
RHD = 1 – sin2  f(x) is not differentiable at x = 2
46. At x = 0, the function y = e–|x| is
(1) Continuous (2) Continuous and differentiable
(3) Differentiable with derivative = 1 (4) Differentiable with derivative = –1
Sol. Answer (1)

From graph, f(x) is continuous but not differentiable at x = 0.


47. Let f(x) =  + |x| + v|x|2, where , , v are real constants, then f (0) exists if
(1)  = 0 (2) v=0 (3) =0 (4) =v
Sol. Answer (1)

f ( x )     | x |  | x |2

⎧⎪  x  x 2 , x  0
f (x)  ⎨
⎪⎩  x  x 2 , x  0

⎧   2x, x  0
f '( x )  ⎨
⎩   2x, x  0

If f'(0) exist then  = 0

{x}
48. If {x} denotes the fractional part of x. Then lim is equal to
x0 tan{ x }

(1) 1 (2) 0 (3) –1 (4) Limit doesn't exist


Sol. Answer (4)
RHL = 1

1
LHL =
tan1

LHL  RHL  limit does not exist.

Aakash Educational Services Pvt. Ltd. Regd. Office : Aakash Tower, 8, Pusa Road, New Delhi-110005 Ph.011-47623456
154 Continuity and Differentiability Solutions of Assignment (Set-2)

49. Let f(x) = [x], g(x) = |x| and f{g(x)} = h(x), where [ · ] is the greatest integer function. Then h(–1) is
(1) 0 (2) – (3) Doesn't exist (4) None of these
Sol. Answer (3)
h(x) = f{g(x)} = [| x |]

⎧ [ x ], x  0
h( x )  ⎨
⎩[  x ], x  0
h(x) is not differentiable at integral point
 h'(–1) does not exist.
50. Let f(x) = max {4, 1 + x2, x2 – 1} xR. Total number of points, where f(x) is non-differentiable, is equal to
(1) 2 (2) 4 (3) 6 (4) 0
Sol. Answer (1)

1 + x2 1 + x2
4

 3 0 3

From the graph of the given function, it follows that f is not differentiable at x   3 .
51. If f is a real-valued differentiable function satisfying |f(x) – f(y)|  (x – y)2, x, y  R and f(0) = 0 then f(1) equals
(1) 1 (2) 2 (3) 0 (4) –1
Sol. Answer (3)
| f(x) – f(y) |  (x – y)2

2
| f (x )  f (y ) | (x  y )
 lim  lim
x y |xy | x  y |xy |

 | f'(y) |  0  | f'(y) | = 0
f'(y) = 0
f(y) = a
f (0) = a = 0 i.e., f(y) = 0,  y  R
Hence f(1) = 0

⎛ sin2 x e x  1 log(1  x ) ⎞
52. The value of lim ⎜   ⎟⎠ is equal to
x 0 ⎝ x x x

(1) 4 (2) 3 (3) 2 (4) 1

Aakash Educational Services Pvt. Ltd. Regd. Office : Aakash Tower, 8, Pusa Road, New Delhi-110005 Ph.011-47623456
Solutions of Assignment (Set-2) Continuity and Differentiability 155

Sol. Answer (1)

⎛ sin 2 x e x  1 log(1  x ) ⎞
lim ⎜   ⎟
x 0 ⎝ x x x ⎠

⎛ sin2 x ⎞ ex  1 log(1  x )
= lim 2 ⎜ ⎟  lim  lim
x  0 ⎝ 2 x ⎠ x 0 x x 0 x

=2+1+1=4

1  cos 2 (cx 2  bx  a )
53. If  and  be the roots of the equation ax2 + bx + c = 0 then lim equals
x
1 4(1  x )2

c ⎛ 1 1⎞ c ⎛ 1 1⎞ c ⎛ 1 1⎞
⎜  ⎟ ⎜  ⎟ ⎜  ⎟
2 ⎜⎝   ⎟⎠
(1) Not possible (2) (3) (4)
2 ⎜⎝   ⎟⎠ 2 ⎜⎝   ⎟⎠

Sol. Answer (3)


 and  are the roots of
ax2 + bx + c = 0
 ax2 + bx + c  a(x – ) (x – ) = 0

⎛ 1 ⎞⎛ 1⎞
and cx2 + bx + a  c ⎜ x  ⎟ ⎜ x  ⎟
⎝  ⎠⎝ ⎠

1  cos2 (cx 2  bx  a )
Now lim
x
1 4(1  x )2

⎛ 1 ⎞⎛ 1⎞
sin2 c ⎜ x  ⎟⎜ x  ⎟ ⎛ 1 ⎞⎛ 1⎞
sin c ⎜ x  ⎟ ⎜ x  ⎟
⎝  ⎠⎝  ⎠ 1 ⎝  ⎠⎝ 
= lim1 = lim ⎠
2
x ⎛1 ⎞ 1 2 ⎛1 ⎞
 4 2 ⎜  x ⎟ x
 ⎜  x⎟
⎝  ⎠ ⎝ ⎠

⎛ 1 ⎞⎛ 1⎞
sin c ⎜ x  ⎟ ⎜ x  ⎟ c ⎛ 1 1⎞
c ⎛ 1⎞ ⎝  ⎠⎝  ⎠ =
= lim ⎜ x ⎟ ⎜  ⎟
1 2 ⎝  ⎛ 1 ⎞ ⎛ 1 ⎞ 2⎝ ⎠
x ⎠ c x x ⎟
 ⎜  ⎟⎠ ⎜⎝ ⎠

2x
⎛ a b ⎞
54. If Lt ⎜1   2 ⎟  e 2 , then the value of a and b are
x  ⎝ x x ⎠

(1) a  R, b  R (2) a = 1, b  R (3) a  R, b = 2 (4) a = 1, b = 2

Aakash Educational Services Pvt. Ltd. Regd. Office : Aakash Tower, 8, Pusa Road, New Delhi-110005 Ph.011-47623456
156 Continuity and Differentiability Solutions of Assignment (Set-2)

Sol. Answer (2)

2x
⎛ a b ⎞
lim ⎜ 1   ⎟  e2
x  ⎝ x x2 ⎠

⎛a b ⎞
lim ⎜  ⎟.2 x
e x  ⎝ x x2 ⎠  e2

2b
lim 2a 
 e x  x  e2
 e2a = e2
 2a = 2  a = 1
a = 1, b  R

⎛ sin x ⎞
(1  sin x )cosec x  e  ⎜ e
⎝ 2 ⎟⎠
55. If the value of lim is k, then (24 k – 11e) is equal to
x 0 sin2 x
(1) 0 (2) 1 (3) 2 (4) 3
Sol. Answer (1)

e sin x
(1  sin x )cosec x  e 
lim 2
x 0 sin2 x

x 1

⎛ sin x ⎞ sin x x e sin x
⎜1  x⎟ e
⎝ x ⎠ 2
= lim 2
x 0
⎛ sin x ⎞ 2
⎜ ⎟ x
⎝ x ⎠

1
1  x  x  e  e sin x
= lim 2
x 0 x2
1
Let y = 1  x  x

1
 log y  log(1  x )
x

1⎛ x2 x3 ⎞
= ⎜x   .... ⎟
x⎝ 2 3 ⎠

x x2 x4
= 1    ....
2 3 4

⎛ 1 x  x 2  x 4 .... ⎞
⎜ ⎟
y = e⎝ 2 3 4 ⎠

Aakash Educational Services Pvt. Ltd. Regd. Office : Aakash Tower, 8, Pusa Road, New Delhi-110005 Ph.011-47623456
Solutions of Assignment (Set-2) Continuity and Differentiability 157

⎛  x  x 2  x 4 .... ⎞
⎜ ⎟
y = e e⎝ 2 3 4 ⎠

⎛  x  x 2  x 4 .... ⎞
⎜ ⎟ e sin x
e e⎝ 2 3 4 ⎠e
2
 Value of the limit = lim
x 0 x2

⎛ x x2 ⎞
⎜   .... ⎟
⎝ 2 3 ⎠ sin x
e  1
= (e ) lim 2
x 0 x2

⎛ x3 x5 ⎞
⎛ ⎛ 2 ⎞ ⎛ 2 ⎞
2 ⎞ ⎜ x   ... ⎟
⎜ 1  ⎜  x  x ... ⎟  1 ⎜  x  x ... ⎟ ... ⎟  1  ⎝ 3! 5! ⎠
⎜ ⎟
⎝ ⎝ 2 3 ⎠ 2⎝ 2 3 ⎠ ⎠ 2
= (e ) lim 2
x 0 x

11e
=
24

11e
k =
24
 24k – 11e = 0

⎧ 1  px – 1– px
⎪ ; –1  x  0
⎪ x
56. f (x)  ⎨ ; is continuous in the interval [–1, 1], then p equals
⎪ 2x  1
; 0  x 1
⎪⎩ x–2

1 1
(1) –1 (2) – (3) (4) 1
2 2
Sol. Answer (2)

⎧ 1  px  1  px
⎪⎪ , 1  x  0
f (x)  ⎨ x
⎪ 2x  1
, 0  x 1
⎪⎩ x 2

For the existence of limit at x = 0 we must have

1  px  1  px 2x  1
lim  lim
x 0  x x 0 x  2

2p 1 1
 lim  p=–
x 0  1  px  1  px 2 2

Aakash Educational Services Pvt. Ltd. Regd. Office : Aakash Tower, 8, Pusa Road, New Delhi-110005 Ph.011-47623456
158 Continuity and Differentiability Solutions of Assignment (Set-2)

⎧a | x 2 – x – 2 |
⎪ 2
,x2
⎪ 2 x – x
57. If f ( x )  ⎪⎨ b ,x2

⎪ x – [x]
,x2
⎪ x–2

is continuous at x = 2 and [ ] is G.I.F., then values of a and b are
(1) a = 1, b = 1 (2) a = 1, b = 2 (3) a = 2, b = 1 (4) None of these
Sol. Answer (1)

⎧a | x2  x  2 |
⎪ 2
, x2
⎪⎪ 2  x  x
f (x)  ⎨ b, x2
⎪ x  [x]
⎪ , x2
⎪⎩ x 2

⎧a, x  2

f ( x )  ⎨b, x  2
⎪ 1, x  2

At x = 2
LHL = a
RHL = 1
f(2) = b
For continuity LHL = RHL = f(2)
a = 1 = b.
58. The function f(x) = sin–1 (cos x) is
(1) Continuous at x = 0 (2) Discontinuous at x = 0
(3) Differentiable at x = 0 (4) None of these
Sol. Answer (1)
We have
f(x) = sin–1 (cos x)

1 ⎛ ⎛ ⎞⎞
= sin ⎜ sin ⎜  x ⎟ ⎟
⎝ ⎝2 ⎠⎠
From graph it is clear f(x) is continuous at x = 0 but not differentiable at x = 0
y


0 
2 x


Aakash Educational Services Pvt. Ltd. Regd. Office : Aakash Tower, 8, Pusa Road, New Delhi-110005 Ph.011-47623456
Solutions of Assignment (Set-2) Continuity and Differentiability 159

1 1 1
59. Let Sn     .....to n terms then lim Sn is equal to
1.4 4.7 7.10 n 

1 1
(1) (2) 3 (3) (4) 
3 4

Sol. Answer (1)

1 1 1 1 1⎡ 1 1 1 1 1 ⎤
Sn    ... = ⎢1     . . .  
1.4 4.7 7.10 n.n  3 3⎣ 4 4 7 n n  3 ⎥⎦

1⎡ 1 ⎤
Sn  1
3 ⎢⎣ n  3 ⎥⎦

1⎡ 1 ⎤ 1
 lim ⎢1
n  3 ⎥⎦
=
n  3 ⎣ 3

n
r
60. Let t r 
1 r  r 2 4
, then lim
n 
∑t
r 1
r
equals

1 1
(1) (2) 1 (3) (4) None of these
4 2
Sol. Answer (3)

r
tr =
1 r  r 4
2

1⎡ 1 1 ⎤

2 ⎢⎣ r 2  1  r r 2  1  r ⎥⎦
t-n =

n
1⎡ 1 1 1 1 1 1 1 1 ⎤
Sn = ∑t
r 1
n  1     
2 ⎢⎣ 3 3 7 7 13 13
...  2  ⎥
n  1  n n 2  n  1⎦

1⎡ 1 ⎤
1
2 ⎣ n  1  n ⎦⎥

= 2

1⎛ 1 ⎞ 1
 S = lim Sn  lim ⎜1  2 ⎟
n  n  2 ⎝ n  1 n ⎠ 2

1
61. Let  and  be real and distinct root of ax2+bx+c=0 then lim [1  sin(ax 2  bx  c )] x  is
x 

(1) e   (2) e  (3) e a (   ) (4) e a(   )

Aakash Educational Services Pvt. Ltd. Regd. Office : Aakash Tower, 8, Pusa Road, New Delhi-110005 Ph.011-47623456
160 Continuity and Differentiability Solutions of Assignment (Set-2)

Sol. Answer (3)


ax2 + bx + c = 0 has roots  and , hence ax2 + bx + c = a(x – )(x – )

1 1
⎢ lim ⎡sin(ax 2  bx c )⎤⎥
lim ⎡1  sin(ax 2  bx  c )⎤ x  = e x  ⎣ ⎦ x 
x  ⎣ ⎦

1 ⎡ sin a( x  )( x ) ⎤
lim  sin a( x  )( x ) lim ⎢a ( x )
x  ⎣ a( x  )( x ) ⎥⎦
= e x  x  = e = e a(  ) .
62. If y = sin–1(3x – 4x3), then the number of points in [–1, 1], where y is not differentiable is
(1) 0 (2) 1 (3) 2 (4) 3
Sol. Answer (3)
Graph of sin–1 3x

1 1
From graph its clear f(x) is not differentiable at x = – and .
2 2


1

-1 1 1


1
63. Suppose f(x) is differentiable at x = 1 and Lt f (1  n )  5, then f (1) equals
n 0 n
(1) 6 (2) 5 (3) 4 (4) 3
Sol. Answer (2)

1
lim f (1  n )  5
n 0 n

f (1  n )
 lim 5
n 0 n

 lim f '(1  n )  5
n 0

 f '(1)  5

64. If f(x) and g(x) be differentiable functions and f(1) = g(1) = 2 then

f (1) · g ( x ) – f ( x ) · g (1) – f (1)  g (1)


Lt is equal to
x 1 g( x ) – f ( x )

(1) 0 (2) 1 (3) 2 (4) None of these

Aakash Educational Services Pvt. Ltd. Regd. Office : Aakash Tower, 8, Pusa Road, New Delhi-110005 Ph.011-47623456
Solutions of Assignment (Set-2) Continuity and Differentiability 161

Sol. Answer (3)

f (1).g ( x )  f ( x ).g (1)  f (1)  g (1)


lim
x 1 g( x )  f ( x )
Applying L. Hospital Rule

f (1)g '( x )  f '( x )g (1)


= lim =2
x 1 g '( x )  f '( x )

⎧ 
⎪ x  a 2 sin x ; 0x
4
⎪⎪  
65. Let f ( x )  ⎨ 2 x cot x  b ; x
⎪ 4 2
⎪a cos 2 x  b sin x ;   x  
⎪⎩ 2
2
a
continuous function x [0, ] then the value of 5⎛⎜ ⎞⎟ is
⎝b⎠
(1) 20 (2) 10 (3) 5 (4) 40
Sol. Answer (1)

⎧ 
⎪ x  a 2 sin x; 0x
4

⎪  
f(x) = ⎨ 2 x cot x  b; x
⎪ 4 2
⎪ 
⎪a cos 2 x  b sin x; 2  x  


at x =
4

 ⎛⎞ 
lim f ( x ) = lim ( x  a 2 sin x ) =  a , and lim f ( x ) = lim 2 x cot x  b , f ⎜ ⎟   b

x 

x  4 
x 

x  ⎝4⎠ 2
4 4 4 4


Since f(x) is continuous at x =
4
⎛⎞
Hence lim f ( x )  lim f ( x )  f ⎜ ⎟

x 

x  ⎝4⎠
4 4

  
 a  b ⇒ab  …(1)
4 2 4

At x = we observe that
2

lim f ( x ) = b, ⎛⎞

lim f ( x ) = –a – b and f ⎜ ⎟  b
x  
x  ⎝2⎠
2 2

Aakash Educational Services Pvt. Ltd. Regd. Office : Aakash Tower, 8, Pusa Road, New Delhi-110005 Ph.011-47623456
162 Continuity and Differentiability Solutions of Assignment (Set-2)

 ⎛⎞
Since f(x) is continuous at x = , hence lim f ( x )  lim f ( x )  f ⎜ ⎟
2 
x  h

x  ⎝2⎠
2 2

 b = –a – b = b
 2b + a = 0 …(2)
On solving (1) and (2), we get


–3b =
4
 
 b and a 
12 6
2
⎛a⎞
Hence 5 ⎜ ⎟  20
⎝b⎠

2 1
9( x 3  2 x 3  1)
66. The value of f(1) so that the function f ( x )  , x  1 is continuous at x  1 is
( x  1)2

(1) 1 (2) 2 (3) 3 (4) 4


Sol. Answer (1)
In order that f(x) is continuous at x = 1, we must have

9( x 2/3  2 x1/3  1)
lim f ( x )  lim = f(1)
x 1 x 1 ( x  1)2

9( x1/3  1)2
 lim  f (1)
x 1 ( x1/3  1)2 ( x 2/3  1  x1/3 )2

 f(1) = 1.

⎛ x 1⎞
67. The value of lim 48 x ⎜  tan 1 ⎟ is
x  ⎝4 x 2⎠

(1) 12 (2) 18 (3) 24 (4) 32


Sol. Answer (3)

⎛ x 1⎞
lim 48 x ⎜  tan1
x  ⎝4 x  2 ⎟⎠

⎛ 1 1 ⎛ x  1 ⎞ ⎞
= lim 48 x ⎜ tan (1)  tan ⎜ ⎟⎟
x  ⎝ ⎝ x  2 ⎠⎠

⎡ ⎛ x 1 ⎞⎤
⎢ ⎜1  x  2 ⎟ ⎥
1 ⎝ ⎠⎥
= lim 48 x ⎢tan
x  ⎢ ⎛ x 1 ⎞⎥
⎢ ⎜1  x  2 ⎟ ⎥
⎣ ⎝ ⎠⎦

Aakash Educational Services Pvt. Ltd. Regd. Office : Aakash Tower, 8, Pusa Road, New Delhi-110005 Ph.011-47623456
Solutions of Assignment (Set-2) Continuity and Differentiability 163

1
1 tan1
= lim 48 x tan
1
= lim 48 x 2 x 3  1
x  2x  3 x  1 2x  3
2x  3

⎛ 1 ⎞
48 x ⎜ tan1 ⎟
1 2 x  3
= lim ⎜∵ lim  0,  1 as x   ⎟
x  2 x  3 ⎜ x  2 x  3 1 ⎟⎟

⎝ 2x  3 ⎠

48 x 48
= lim = = 24
x  x (2  3 / x ) 2
68. Let f(x) be a continuous function defined for  x  R. If f(x) take rational values  x  R and f(2) = 198,
then f(22) = ......
(1) 99 (2) 198 (3) 33 (4) 4
Sol. Answer (2)
As f(x) is continuous function in [1, 3] and takes rational value only hence f(x) is a constant function.
 f(x) = f(2) = 198
 f(22) = 198

sin2 x  sin2 y
69. If lim  g ( y ) and the limiting value of lim g ( y )  k , then the value of 1 + k + k2 + k3 + …… +
x y x2  y 2 y 0

2010
k is
(1) 2011 (2) 2010 (3) 2009 (4) 2008
Sol. Answer (1)

sin2 x  sin2 y
Given g(y) = lim
x y x2  y 2

sin( x  y ) sin( x  y )
= xlim 
y (x  y ) (x  y )

sin( x  y ) sin( x  y )
= xlim  lim
y (x  y ) hy ( x  y )

sin2y sin2y
= 1 =
2y 2y

sin2y
Now, k  lim 1
y 0 2y
 1 + k + k2 + k3 + ……+ k2010
= 1 + 1 + 1 + 1 + …… (2011) times = 2011

Aakash Educational Services Pvt. Ltd. Regd. Office : Aakash Tower, 8, Pusa Road, New Delhi-110005 Ph.011-47623456
164 Continuity and Differentiability Solutions of Assignment (Set-2)

x  x 2    x n  n
70. Let f ( x )  , g(x) = (4n + 5n)1/n and  and  are the roots of equation lim f ( x )  nlim

g( x ) ,
x 1 x 1

 n
⎛ 1 1⎞
then the value of
n  0⎝
 ⎠∑
⎜  ⎟ is

5 10 10 5
(1) (2) (3) (4)
9 3 9 3

Sol. Answer (3)

Given lim f ( x )  lim g ( x )


n 1 n 

n(n  1)
 5
2
 n2 + n – 10 = 0
  +  = – 1,  = – 10

1 1 1
Now,  
  10

 n
⎛ 1 1⎞
 ∑ ⎜  ⎟
n 0 ⎝
 ⎠

2
1 ⎛ 1 ⎞
= 1    to 
10 ⎜⎝ 10 ⎟⎠

1
10
= 1 1 =
10 9

sin 2 x  a sin x
71. If lim  b (finite), then the value of (ab)2 is
x 0 x3
(1) 1 (2) 2 (3) 3 (4) 4
Sol. Answer (4)

sin2 x  a sin x
lim b
x 0 x3

⎛ (2 x )3 (2 x )5 ⎞ ⎛ x3 x5 ⎞
⎜ 2x    ... ⎟  a ⎜ x   ... ⎟
⎜ 3! 5! ⎟ ⎜ 3! 5! ⎟⎠
 lim ⎝ ⎠ ⎝ b
x 0 x3

Aakash Educational Services Pvt. Ltd. Regd. Office : Aakash Tower, 8, Pusa Road, New Delhi-110005 Ph.011-47623456
Solutions of Assignment (Set-2) Continuity and Differentiability 165

⎛ 8 a⎞ ⎛ 32 a ⎞
x (a  2)  x 3 ⎜   ⎟  x 5 ⎜  ⎟  ...
 ⎝ 6 6 ⎠ ⎝ 120 120 ⎠
lim b
x 0 x3

 a20  a = –2

8 a
  b  b = –1
6 6
 (ab)2 = 22 = 4.

⎡ 2008 sin x ⎤ ⎡ 2009 tan x ⎤


72. The value of lim ⎢
x 0 ⎣ x ⎥⎢ x ⎥ , (where [x] denotes the greatest integer  x) is
⎦ ⎣ ⎦
(1) 4015 (2) 4016 (3) 4017 (4) 4018
Sol. Answer (2)

⎡ ⎛ sin x ⎞ ⎤ ⎡ ⎛ tan x ⎞ ⎤
lim ⎢ 2008 ⎜
x ⎟ ⎥  lim ⎢ 2009 ⎜ x ⎟ ⎥
x 0 ⎣ ⎝ ⎠⎦ x  0 ⎣ ⎝ ⎠⎦

As we know that, if x > 0, sin x < x and tan x > x using this concept

⎡ sin x ⎤ ⎡ tan x ⎤
lim ⎢2008 ⎥  lim ⎢2009
x ⎥⎦
= 2007 + 2009 = 4016.
x 0 ⎣ x ⎦ x 0 ⎣

x 1

73. If Lt 1  ax  bx 2  x 1  e3 , then the value of a + b + bc + 2013 is

(1) 2013 (2) 2014 (3) 2015 (4) 2016


Sol. Answer (4)
c
Given Lt (1  ax  bx 2 ) x 1  e3
x 1

( ax  bx 2 )c
Lt
 e x 1 x 1  e3 (∵ a  b  0)

(ax  bx 2 )c
 Lt 3
x 1 x 1

(a  2b )c
 Lt 3
x 1 1
 ac +2bc = 3
 ac + bc + bc = 3
 c(a + b) + bc = 3
 bc = 3 (∵ a + b = 0)
 a + b + bc + 2009
= 3 + 2009 = 2012

Aakash Educational Services Pvt. Ltd. Regd. Office : Aakash Tower, 8, Pusa Road, New Delhi-110005 Ph.011-47623456
166 Continuity and Differentiability Solutions of Assignment (Set-2)

⎛x⎞ ⎛x⎞ ⎛ x ⎞ ⎛ 1  (g ( x ))2011 ⎞


74. Let f(x) = cos ⎜ ⎟  cos ⎜ ⎟  cos ⎜ n ⎟ . If lim f ( x )  g ( x ) , then the value of lim ⎜ ⎟ is
2
⎝ ⎠ 4
⎝ ⎠ ⎝2 ⎠ n  x 0
⎝ 1 g( x ) ⎠
(1) 2010 (2) 2011 (3) 2012 (4) 2013
Sol. Answer (2)

⎛ x ⎞ ⎛ x ⎞ ⎛ x ⎞ ⎛x⎞
Given, f(x) = cos ⎜ n ⎟  cos ⎜ n 1 ⎟ cos ⎜ 2 ⎟  cos ⎜ 2 ⎟
⎝2 ⎠ ⎝2 ⎠ ⎝2 ⎠ ⎝ ⎠

x
= cosA.cos2A.cos22A……cos2n–1A, A  (say)
2n

⎛ ⎛ x ⎞⎞
n sin ⎜ 2n  ⎜ n ⎟⎟
sin 2 ( A) ⎝ ⎝2 ⎠⎠
= n =
2  sin A ⎛ x ⎞
2n  sin ⎜ n ⎟
⎝2 ⎠

⎛ x ⎞
sin x ⎜ n⎟
2 ⎠
=  ⎝
x ⎛ x ⎞
sin ⎜ n ⎟
⎝2 ⎠

⎛ x ⎞
sin x ⎜ n⎟ sin x sin x
2 ⎠
 lim f ( x )  lim  lim ⎝ = 1=  g( x )
n  n  x n  ⎛ x ⎞ x x
sin ⎜ n ⎟
⎝2 ⎠

2010
⎛ ⎛ sin x ⎞
2011 ⎞
⎛ sin x ⎞ d ⎛ sin x ⎞
⎜ 1 ⎜ ⎟ ⎟ 2011  ⎜ ⎟  ⎜ ⎟
lim ⎜ ⎝ x ⎠ ⎟ = lim ⎝ x ⎠ dx ⎝ x ⎠
 = 2011
x 0 ⎜ ⎛ sin x ⎞ ⎟ x 0 d ⎛ sin x ⎞
⎜ 1 ⎜ ⎟ ⎟ ⎜
dx ⎝ x ⎠

⎝ ⎝ x ⎠ ⎠

⎧ p ⎛ 1⎞
⎪ x sin ⎜ ⎟ , x  0
75. Let f ( x )  ⎨ ⎝x⎠ and A and B are two sets of values of p such that A and B are continuous
⎪ 0 , x 0

and differentiable of f(x) at x = 0 respectively then A B is

(1) (1,  ) (2) (2,  ) (3) ( , 1) (4) ( , 0)


Sol. Answer (1)

⎛ 1⎞
A  lim x p sin ⎜ ⎟
x 0 ⎝x⎠
= 0, when p > 0

Aakash Educational Services Pvt. Ltd. Regd. Office : Aakash Tower, 8, Pusa Road, New Delhi-110005 Ph.011-47623456
Solutions of Assignment (Set-2) Continuity and Differentiability 167

 A = (0, )

⎛ 1⎞
x p sin ⎜ ⎟  0
 B  lim ⎝x⎠
x 0 x 0

⎛ 1⎞
= xlim x p 1 sin ⎜ ⎟
0 ⎝x⎠
= 0, when p – 1 > 0
 B = (1, )
 A  B = {(0, )  (1, )} = (1, )

Differentiation

1 1 1 dy bc
76. If y  a b c b
 b c a c
 b a c a then find at e a .
1 x x 1 x x 1 x x dx
(1) 0 (2) 1 (3) 2 (4) 3
Sol. Answer (1)

1 1
y   ...
a b c b b c
1 x x 1 x  x a c

xb xc xa
y  
xa  xb  xc xa  xb  xc xa  xb  xc

xa  xb  xc
y 1
xa  xb  xc

dy
0
dx
77. If f(x) = log |x| then for x  0, f(x) is equal to

1 1 1 1
(1) | x | (2) (3)  (4) 
x x x
Sol. Answer (2)

d 1 d
f ( x )  (log | x |) = |x|
dx | x | dx

1 x x 1
= | x || x | = = ,x0
| x |2 x

d ⎛ 1 2 x ⎞
78. sin ⎟ is equal to
dx ⎜⎝ 1 x2 ⎠

2 2 3(1  x 2 ) 2
(1) (2)  (3) 2 2
, x  1 (4)
1 x 2
1 x 2 | 1  x | (1  x ) 1 x2

Aakash Educational Services Pvt. Ltd. Regd. Office : Aakash Tower, 8, Pusa Road, New Delhi-110005 Ph.011-47623456
168 Continuity and Differentiability Solutions of Assignment (Set-2)

Sol. Answer (1)


Put x = tan 

2 tan 
 f ( x )  sin1  2  2 tan1 x
1  tan2 

2
 f ( x ) 
1 x2
79. Differential coefficient of log10 x w.r.t. logx 10 is

(log x )2 (log10 x )2 (logx 10)2 (log10)2


(1)  (2) (3) (4) 
(log10)2 (log10)2 (log10)2 (log x )2

Sol. Answer (1)


If y = log10x z = logx10

log x log10
y z
log10 log x

d ⎛ log x ⎞ ⎛ 1 ⎞ 1
⎜ ⎟⎜ ⎟
dy dx ⎝ log10 ⎠⎝ log10 ⎠ x
 
dz d ⎛ log10 ⎞ (log10)  1  1
⎜ ⎟ (log x )2 x
dx ⎝ log x ⎠

(log x )2
= 
(log10)2

⎡ 3⎤
⎢ x ⎛ x  2 ⎞4 ⎥ dy
80. If y  log ⎢e ⎜
x  2 ⎟ ⎥ then dx 
⎝ ⎠
⎢⎣ ⎥⎦

x2  4 3 x2  1 3x 2
(1) (2) (3) (4)
x2  1 x2  4 x2  4 x2  4
Sol. Answer (3)

⎛ 3 ⎞
⎜ x ⎛ x  2 ⎞4 ⎟
y  log ⎜ e ⎜ ⎟ ⎟
⎜ ⎝ x 2⎠ ⎟
⎝ ⎠

3
x
= log e  log( x  2)  log( x  2)
4

3
= x log( x  2)  log( x  2)
4

Aakash Educational Services Pvt. Ltd. Regd. Office : Aakash Tower, 8, Pusa Road, New Delhi-110005 Ph.011-47623456
Solutions of Assignment (Set-2) Continuity and Differentiability 169

dy 3⎡ 1 1 ⎤
 1 ⎢ 
dx 4 ⎣ x  2 x  2 ⎥⎦

x2  1
=
x2  4

e x  ex dy
81. If y  x x
, then 
e e dx

(1) 1 + y2 (2) 1 – y2 (3) y2 – 1 (4) y2 + 2


Sol. Answer (2)

dy (e x  e  x )(e x  e  x )  (e x  e  x )(e x  e  x )
 = 1 – y2
dx (e x  e  x )2

d 2y
82. If y = aemx + be–mx, then 
dx 2
(1) m2y (2) –m 2y 2 (3) my (4) –my
Sol. Answer (1)

y1  amemx  b ( m) e mx

y 2  am2emx  b ( m)2 e mx  m2 y

d 2y
83. If y2 = ax2 + b, then 
dx 2

ab x3 ab ab
(1) (2) (3) 2 (4)
x 3
ab y y3

Sol. Answer (4)


y2 = ax2 + b

dy dy ax
2y  2ax ⇒ 
dx dx y

dy ay  ax ⎛ ax ⎞
2 ay  ax ⎜ y ⎟
d y
 dx  ⎝ ⎠  ab
dx 2 y2 y2 y3

log x d 2y
84. If y  , then 
x dx 2

3  2log x 2log x  3 2log x  3 2  3log x


(1) 3 (2) 3 (3) 4 (4)
x x x x4

Aakash Educational Services Pvt. Ltd. Regd. Office : Aakash Tower, 8, Pusa Road, New Delhi-110005 Ph.011-47623456
170 Continuity and Differentiability Solutions of Assignment (Set-2)

Sol. Answer (2)

⎛ 1⎞
x  log x
d ⎛ log x ⎞ ⎜⎝ x ⎟⎠ 1  log x
y1  ⎜ ⎟  2

dx ⎝ x ⎠ x x2

⎛ 1⎞
x 2 ⎜  ⎟  (1  log x )2 x
⎝ x⎠ 2log x  3
y2  2 2

(x ) x3

85. The differential coefficient of f(loge x) w.r.t. x, where f(x) = loge x, is

x 1 1
(1) (2) loge x (3) (4) x log x
loge x x x loge x

Sol. Answer (3)

f (loge x )  loge (loge x )

df (loge x ) 1 1
  
dx loge x x

n dy
86. If y = (1 + x) (1 + x2) (1 + x4)..... (1  x 2 ) then at x = 0 is
dx
(1) 1 (2) –1 (3) 0 (4) 2
Sol. Answer (1)
n n 1
(1  x )(1  x )(1  x 2 )(1  x 4 ).....(1  x 2 ) 1  x 2
y 
(1  x ) 1 x

n 1 n 1
dy 2n  1  x 2 1
 (1  x )  1  x 2
 
dx (1  x )2

at x = 0

dy 2n  1  0  1  1  0
   1.
dx 12


87. If f(x) = cos x cos 2x cos 4x cos 8x cos 16x then f  ⎛⎜ ⎞⎟ is
⎝4⎠

1
(1) 2 (2)
2
(3) 1 (4) –1

Sol. Answer (1)

2 sin x cos x cos 2 x cos 4 x cos8 x cos16 x sin32x


f(x) = = 5
2 sin x 2 sin x

Aakash Educational Services Pvt. Ltd. Regd. Office : Aakash Tower, 8, Pusa Road, New Delhi-110005 Ph.011-47623456
Solutions of Assignment (Set-2) Continuity and Differentiability 171

1 32cos32x sin x  cos x sin32 x


f'(x) =
32 sin2 x

1 1
32   (0)
⎛⎞ 2 2
f ⎜ ⎟  2
⎝4⎠ ⎛ 1 ⎞
32  ⎜ ⎟
⎝ 2⎠

dy 5
88. If y = cos–1 (cos x) then at x  is equal to
dx 4

1
(1) 1 (2) –1 (3) (4) 2
2
Sol. Answer (2)
1
y = cos cos(   x  ) = cos1 cos(   x  )

5
When x is around ,   x   is in second quadrant.
4
so y =  – (x – )

dy
  1.
dx

y  e y ..... dy
89. If x  e y  e then is
dx

x 1 1 x x
(1) (2) (3) (4)
1 x x x 1 x
Sol. Answer (3)
x = ey + x
log x = y + x

1 dy
 1
x dx

dy
90. If xy . yx = 16 then at (2, 2) is equal to
dx
(1) 1 (2) –1 (3) 0 (4) 2
Sol. Answer (2)
y log x + x log y = log 16

y dy x dy
 (log x )  (log y )  0
x dx y dx

Aakash Educational Services Pvt. Ltd. Regd. Office : Aakash Tower, 8, Pusa Road, New Delhi-110005 Ph.011-47623456
172 Continuity and Differentiability Solutions of Assignment (Set-2)

dy
1  (1) 0
dx

dy
  1.
dx (2,2)

dy
91. If y = sin x0 and z = cos x, then is equal to
dz

 x
(1) –cosec x cos x (2) cosec cos x
180 180

 x  cos x
(3)  cos ec x cos (4) sin x
180 180 180 180
Sol. Answer (3)

x
y = sin ; z = cos x
180

x 
cos 
dy
 180 180   cos x (  cosec x ).
dz  sin x 180 180

92. Let the function y = f(x) be given by


x = t5 – 5t3 – 20t + 7
and y = 4t3 – 3t2 – 18t + 3,
where t (–2, 2) then f(x) at t = 1 is

5 2 7 5
(1) (2) (3) (4)
2 5 5 7
Sol. Answer (2)

dy 12t 2  6t  18 12  6  18 2
   .
dx t 1
4 2
5t  15t  20 t  1 5  15  20 5

du
93. If u = f(x3); v = g(x2); f(x) = cos x and g(x) = sin x is
dv

3 2 3
(1) x cos x 3 cosec x 2 (2) sin x 3 sec x 2 (3) tan x (4) x sin x 2 sec x 3
2 3 2
Sol. Answer (1)

du
du dx f ( x 3 )  3 x 2 cos x 3  3 x 2 3
    x cos x 3 cosec x 2 .
dv dv g ( x 2 )  2 x sin x 2 .2 x 2
dx

Aakash Educational Services Pvt. Ltd. Regd. Office : Aakash Tower, 8, Pusa Road, New Delhi-110005 Ph.011-47623456
Solutions of Assignment (Set-2) Continuity and Differentiability 173

1 1
94. The differential coefficient of cosec 1 2
w.r.t. 1  x 2 at x  is equal to
2x  1 2
(1) –4 (2) 4 (3) –1 (4) –2
Sol. Answer (1)
x = cos 

1
y = cosec 1 = cosec–1 sec 2
2
2x  1

1 ⎧ ⎛ ⎞⎫ 
= cosec ⎨cosec ⎜  2 ⎟ ⎬   2
⎩ ⎝2 ⎠⎭ 2

z= 1  cos2   sin 

dy
dy d  2 2
   
dz dz cos  x
d

2
  4.
x x1
2

dy
95. If t(1 + x2) = x and x2 + t2 = y then at x = 2 is
dx

88 488 244
(1) (2) (3) 1 (4)
125 125 125
Sol. Answer (2)

x
t
1 x2

dt dy
and 2 x  2t 
dx dx

dt (1  x 2 )  x (2 x ) 1 x2
  
dx (1  x 2 )2 (1  x 2 )2

dy 2x 1 x2
  2x  
dx 1  x 2 (1  x 2 )2

Now, put x = 2

dy 488
 
dx 125

Aakash Educational Services Pvt. Ltd. Regd. Office : Aakash Tower, 8, Pusa Road, New Delhi-110005 Ph.011-47623456
174 Continuity and Differentiability Solutions of Assignment (Set-2)

d2x ⎛ ⎞
96. x = t cos t, y = t + sin t then 2
at ⎜ t  ⎟ is equal to
dy ⎝ 2⎠

4 4
(1) (2)  (3) –2 (4) 2
2 2
Sol. Answer (2)

dx
dx cos t  t sin t
 dt 
dy dy 1  cos t
dt

d ⎛ dx ⎞ ( 2 sin t  t cos t )(1  cos t )  (cos t  t sin t )(  sin t )


2 ⎜ ⎟
d x dt ⎝ dy ⎠ (1  cos t )2
  
dy 2 dy 1  cos t
dt

 d2x 4
Now, Put t   2
 .
2 dy 2

d 6y 
97. If y = sin 2x, then 6 at x  is equal to
dx 2

(1) –64 (2) 0 (3) 64 (4) 128


Sol. Answer (2)

dy ⎛ ⎞
 2cos 2 x  2sin ⎜ 2 x  ⎟
dx ⎝ 2⎠

d 2y ⎛ ⎞ ⎛ ⎞
 22 cos ⎜ 2 x  ⎟  22 sin ⎜ 2 x  2 ⎟
2
dx ⎝ 2⎠ ⎝ 2⎠

d 6y ⎛ ⎞
 26 sin ⎜ 2 x  6  ⎟ = 26 sin (2x + 3)
6
dx ⎝ 2⎠

d6y 
 0 at x 
dx 6 2

x3 sin x cos x
d3
98. Let f ( x )  6 1 0 where p is a constant. Then 3
f (x) is
2 3
dx x 0
p p p

(1) p (2) p + p2 (3) p + p3 (4) Independent of p

Aakash Educational Services Pvt. Ltd. Regd. Office : Aakash Tower, 8, Pusa Road, New Delhi-110005 Ph.011-47623456
Solutions of Assignment (Set-2) Continuity and Differentiability 175

Sol. Answer (4)


3 3 3 2
f(x) =  p x  6 p sin x  (6 p  p )cos x

3 2 3 2
f'(x) = 3 p x  6 p cos x  (6 p  p)sin x

f''(x) = 6 p3 x  6 p3 sin x  (6 p2  p )cos x

3 3 2
f'''(x) = 6 p  6 p cos x  (6 p  p)sin x

f'''(0) = 6 p3  6 p3  0 (independent of p).

d 2y 1
99. If y = at2, x = 2at, where a is a constant, then 2 at x  is equal to
dx 2

1
(1) 8a (2) 1 (3) 2a (4)
2a

Sol. Answer (4)


The given function can be written as
x2 = 4ay

dy x d 2y 1
  ⇒ 2

dx 2a dx 2a

1
dy
100. If y  x x , the value of at x = e is equal to
dx

⎛ 1⎞
⎜ ⎟2
(1) 1 (2) 0 (3) –1 (4) e⎝ e ⎠

Sol. Answer (2)

1
y=
xx

1
log y = log x
x

1 dy 1 ⎛ 1 ⎞
= 2  log x  ⎜ 2 ⎟
y dx x ⎝x ⎠

1 dy 1 ⎛ 1 ⎞
= 2  (1) ⎜ 2 ⎟  0
y dx e ⎝e ⎠

dy
 0.
dx

Aakash Educational Services Pvt. Ltd. Regd. Office : Aakash Tower, 8, Pusa Road, New Delhi-110005 Ph.011-47623456
176 Continuity and Differentiability Solutions of Assignment (Set-2)

1 x 1 dy
101. If y  tan for |x| > 1 then =
x 1 dx

1 1 1 1
(1) (2) (3) (4)
2
2 | x | x 1 2
2x x  1 2x x 2  1 2 | x | x2  1

Sol. Answer (1)


Let x = sec 

1 sec   1
y = tan
sec   1

1 1  cos 
= tan
1  cos 

1 ⎛⎞
= tan cot ⎜ ⎟
⎝2⎠

 1
=  sec 1 x
2 2

dy 1
 = 
dx 2 | x | x2  1

dy
102. If y = log2 loge x then is equal to
dx

1 1 1 1
(1) log2 e  logx e (2) log2 x (3) loge x (4) logx e
x x x x
Sol. Answer (1)

⎛ 1⎞ 1

loge loge x ⎜⎝ x ⎟⎠ loge x 1
y   log2 e logx e.
loge 2 loge 2 x

dy
103. If f ( x )  2 x 2  1 and y = f(x2) then dx is equal to
x 1

(1) 2 (2) 1 (3) –2 (4) –1


Sol. Answer (1)

dy 2 4
y = f(x2)  = f ( x )2 x = (2 x ) 2x  1
dx

dy
At x = 1, = 2  1 2  1
dx

Aakash Educational Services Pvt. Ltd. Regd. Office : Aakash Tower, 8, Pusa Road, New Delhi-110005 Ph.011-47623456
Solutions of Assignment (Set-2) Continuity and Differentiability 177

ex  1 y 2 dy
104. If y  , then  is equal to
ex  1 2 dx

1 1
(1) 1 (2) –1 (3)  (4)
2 2
Sol. Answer (4)

ex  1
y=
ex  1

dy (e x  1) e x  (e x  1) e x
=
dx (e x  1)2

e x (e x  1  e x  1) 2e x
= = 
(e x  1)2 (e x  1)2

y 2 dy (e x  1)2 2e x
  = 
2 dx 2(e x  1)2 (e x  1)2

(e x  1)2  4e x
=
2(e x  1)2

(e x  1)2
=
2(e x  1)2

105. If f(x) = ex g(x), g(0) = 2, g(0) = 1, then f(0) =


(1) 1 (2) 3 (3) 2 (4) 0
Sol. Answer (2)
f(x) = ex g(x)
f'(x) = ex g(x) + ex g'(x)
f'(0) = e0 g(0) + e0 g'(0) = g(0) + g'(0)
= 2 + 1 = 3.

d 2y
106. If ax2 + 2hxy + by2 = 1, then is equal to
dx 2

h 2  ab h2  ab ab  h 2 ab  h2
(1) (2) (3) (4)
(hx  by )2 (hx  by )3 (hx  by )3 (hx  by )2

Sol. Answer (2)


ax2 + 2hxy + by2 = 1

⎛ dy ⎞ dy
 2ax  2h ⎜ x  y ⎟  2by 0
⎝ dx ⎠ dx

Aakash Educational Services Pvt. Ltd. Regd. Office : Aakash Tower, 8, Pusa Road, New Delhi-110005 Ph.011-47623456
178 Continuity and Differentiability Solutions of Assignment (Set-2)

dy ax  hy
= 
dx hx  by

⎛ dy ⎞ ⎛ dy ⎞
(hx  by ) ⎜ a  h ⎟  (ax  hy ) ⎜ h  b
d 2y ⎝ dx ⎠ ⎝ dx ⎟⎠
= 
dx 2 (hx  by )2

dy
(ahx  aby  ahx  h 2 y )  (h 2 x  hby  abx  hby )
=  dx
(hx  by )2

dy
y (ab  h2 )  x(h2  ab )
=  dx
(hx  by )2

⎛ ax  hy ⎞
y (ab  h2 )  x(h2  ab ) ⎜ ⎟
 ⎝ hx  by ⎠
=
(hx  by )3

(hyx  by 2 )(ab  h2 )  (h2  ab )(ax 2  hxy )


= 
(hx  by )3

(ab  h2 )(hxy  by 2  ax 2  hxy )


= 
(hx  by )3

h2  ab
= (∵ ax2 + 2hxy + by2 = 1 )
(hx  by )3

107. Derivative of the function f ( x )  log5 (log7 x ) , where x > 7 is

1 1
(1) (2)
x (log5)(log7)(log7 x ) x (log5)(log7)

1 1
(3) (4)
x(loge x )(log7 5) (log5)(log7)( x log x )

Sol. Answer (3)


Given, y = log5 log7 x

loge x
y = log5
loge 7

= log5 loge x  log5 loge 7

loge loge x
y=  log5 loge 7
loge 5

dy 1 ⎡ 1 1⎤
 = ⎢  ⎥
dx loge 5 ⎣ loge x x ⎦

Aakash Educational Services Pvt. Ltd. Regd. Office : Aakash Tower, 8, Pusa Road, New Delhi-110005 Ph.011-47623456
Solutions of Assignment (Set-2) Continuity and Differentiability 179

108. A function f is defined by f(x2) = x3 x > 0 then f (4) equals


(1) 1 (2) 2 (3) 3 (4) Not differentiable
Sol. Answer (3)

f ( x2 )  x3

Differentiating it w.r.t. x we get

2 xf '( x 2 )  3 x 2

3x2 3x
 f '( x 2 )  =
2x 2

32
 f'(4) = =3
2

dy
109. If 3sinxy + 4cosxy = 5, then equals
dx

y y x x
(1) (2)  (3) (4) 
x x y y

Sol. Answer (2)


We have
3sin xy + 4cos xy = 5

⎡ dy ⎤ ⎡ dy ⎤
 3cos xy ⎢ x  y ⎥  4 sin xy ⎢ x  y⎥  0
⎣ dx ⎦ ⎣ dx ⎦

⎡ dy ⎤
 (3cos xy – 4sin xy) ⎢ x  y⎥  0
⎣ dx ⎦

xdy dy  y
  y  
dx dx x

1
110. Let g(x) be the inverse of the function f(x), and f ( x )  then g(x) equals
1 x 3

1 1
(1) (2) (3) 1 + g3 (4) 1 + f3
(1  g )3
(1  f 3 )

Sol. Answer (3)

1
f'(x) =
1 x3
f –1(x) = g(x)
 f(g(x)) = x

Aakash Educational Services Pvt. Ltd. Regd. Office : Aakash Tower, 8, Pusa Road, New Delhi-110005 Ph.011-47623456
180 Continuity and Differentiability Solutions of Assignment (Set-2)

Differentiating it w.r.t. x we get


[f'(g(x))]g'(x) = 1
1 1
g '( x )  =
f '(g ( x )) 1
1 g3
 g'(x) = 1 + (g(x))3
111. Let F(x) = f(x).g(x).h(x) xR and f(x), g(x), h(x) are differentiable functions at some point x0R.
F(x0) = 21 F(x0), f (x0) = 4f(x0)
g(x0) = –7g(x0) and h(x0) = kh(x0).
Then 3k + 28 is equal to
(1) 20 (2) 50 (3) 100 (4) 200
Sol. Answer (3)
F(x) = f(x).g(x).h(x)
 F'(x) = f'(x).g(x).h(x) + f(x ).g'(x.h(x)) + f(x).g(x).h'(x)
 F'(x0) = f'(x0).g(x0).h(x0) + f(x0).g'(x0).h(x0) + f(x0).g(x0).h'(x0)
 21F(x0) = 4 f(x0).g(x0).h(x0) – 7 f(x0).g(x0).h(x0) + k f(x0).g(x0).h(x0)
 21F(x0) = (–3 + k) F(x0), k = 24
Hence 3k + 28 = 100.

Mean Value Theorem and Functional Equation


112. Rolle’s theorem is not applicable to f(x) = |x| in [–2, 2] because
(1) f(x) is not continuous in [–2, 2] (2) f(x) is not derivable in (–2, 2)
(3) f(2)  f(–2) (4) It is applicable
Sol. Answer (2)
f(x) = |x| is continuous in [–2, 2]
Also, f (2) = f(–2) = 2, but f(x) is not
derivable at x = 0
113. Lagrange’s mean value theorem is not applicable to f(x) in [1, 4] where f(x) =
(1) x2 – 2x (2) |x – 2| (3) x |x| (4) x3
Sol. Answer (2)
f(x) = |x – 2| is not derivable at x = 2  (1, 4)
Hence, LMVT is not applicable to f(x) = |x – 2| in [1, 4].
114. The value of c in Lagrange’s mean value theorem for the function f(x) = |x| in the interval [–1, 1] is
1
(1) 0 (2)
2
1
(3)  (4) Non-existent in the interval
2

Aakash Educational Services Pvt. Ltd. Regd. Office : Aakash Tower, 8, Pusa Road, New Delhi-110005 Ph.011-47623456
Solutions of Assignment (Set-2) Continuity and Differentiability 181

Sol. Answer (4)


f (1)  f ( 1)
By Lagrange’s  f (c )
1  ( 1)
i.e., 0 = f'(c) where – 1 < c < 1
If x > 0, f(x) = x therefore f'(x) = 1 and
If x < 0, f(x) = –x therefore f'(x) = –1
Hence, there is no c  0 for which 0 = f'(c) holds. Also at x = 0, f(x) is not differentiable.
Hence, c has no value.
115. The value of c according to Rolle’s theorem for the function f(x) = x3 – 9x2 + 26x – 24 in the interval [2, 4] is

9 3
(1) 9  3 (2) 9 3 (3) (4) 9 3
3
Sol. Answer (3)
f(x) is a polynomial
 It is continuous in [2, 4] and is differentiable in (2, 4) and f(2) = 0, f(4) = 0
 Rolle’s theorem is applicable.
 f'(c) = 0 c  [2, 4]

f ( x )  3 x 2  18 x  26

3c 2  18c  26  0

18  12 9  3
 c 
6 3
116. The value of c according to Lagrange’s mean value theorem for the function f(x) = 2x2 – 10x + 29 in the interval
[2, 7] is
(1) 4.5 (2) 5 (3) 6 (4) 4
Sol. Answer (1)
f(x) being a polynomial is continuous and differentiable in the given interval

f ( b )  f (a )
 f (c ) 
ba
40 9
4c  10  ⇒ c   4.5
5 2
117. If f(x) = x ·logx and f(0) = 0, then the value of  for which Rolle’s theorem can be applied in [0, 1] is
1
(1) –2 (2) –1 (3) 0 (4)
2
Sol. Answer (4)
f(x) = x log x cannot be defined for  = –2, –1
for  = 0 f(x) = log x f(0)  0
1
  is only possibility
2

Aakash Educational Services Pvt. Ltd. Regd. Office : Aakash Tower, 8, Pusa Road, New Delhi-110005 Ph.011-47623456
182 Continuity and Differentiability Solutions of Assignment (Set-2)

118. In [0, 1], Lagrange mean value theorem is NOT applicable to

⎧ 1 1
⎪ 2  x, x
2 ⎧ sin x
⎪ ⎪ , x0
(1) f ( x )  ⎨ 2 (2) f (x)  ⎨ x
⎪⎛ 1  x ⎞ , x  1 ⎪⎩ 1, x 0

⎪⎩⎝ 2 ⎟ 2

(3) f(x) = x|x| (4) f(x) = |x|


Sol. Answer (1)
119. The value of C (if exists) in Lagrange's theorem for the function |x| in the interval [–1, 1] is

1
(1) 0 (2)
2

1
(3) – (4) Non-existent in the interval
2
Sol. Answer (4)
| x | is not differentiable at x = 0 hence conditions of Lagrange’s theorem is not satisfied consequently c does
not exist.
120. Let f(x + y) = f(x). f(y) x, y R. Suppose that f(k) = 3, k  R and f (0) = 11, then f (k) is equal to
(1) 3 (2) 11 (3) 33 (4) 0
Sol. Answer (3)
We have,
f(x + y) = f(x). f(y) x, y  R
f(3) = 3, f'(0) = 11
Let us put x = 0, y = 0 in f(x + y) = f(x). f(y)
 f(0) = f(0)2
 f(0) = 1 as f(0) = 0
 f(x) = 0,  x  R f(3) = 0,  3, a contradiction

f ( x  h)  f ( x ) f ( x ).f ( h )  f ( x )
 f'(x) = lim = lim
h 0 h h 0 h

f ( x )[f (h )  1] f ( x )[f (h )  f (0)]


= lim = lim = f(x) f'(0)
h 0 h h 0 h

 f'(x) = f(x) f'(0)


 f'(3) = f(3) f'(0) = 3 × 11 = 33
121. Let f(x) is a polynomial satisfying f(x).f(y) = f(x) + f(y) + f(xy) – 2  x, y and f(2) = 1025, then the value
of lim f ( x ) is
x 2

(1) 5  28 (2) 5  29 (3) 5  27 (4) 5  210

Aakash Educational Services Pvt. Ltd. Regd. Office : Aakash Tower, 8, Pusa Road, New Delhi-110005 Ph.011-47623456
Solutions of Assignment (Set-2) Continuity and Differentiability 183

Sol. Answer (4)


Given f(x).f(y) = f(x) + f(y) + f(xy) – 2

1
Put y 
x

⎛ 1⎞ ⎛ 1⎞
 f ( x )  f ⎜ ⎟  f ( x )  f ⎜ ⎟  f (1)  2
⎝x⎠ ⎝x⎠
Put x = 1
 {f(1)}2 = 3 f(1) – 2
 {f(1)}2 – 3 f(1) + 2 = 0
 (f(1) – 1) (f(1) – 2) = 0
 f(1) = 1, 2
 f(1) = 2, (∵ f(1)  1)

⎛ 1⎞ ⎛ 1⎞
 f (x)  f ⎜ ⎟  f (x)  f ⎜ ⎟  2  2
x
⎝ ⎠ ⎝x⎠

⎛ 1⎞ ⎛ 1⎞
 f (x)  f ⎜ ⎟  f (x)  f ⎜ ⎟
x
⎝ ⎠ ⎝x⎠

 f(x) = 1 ± xn
 f(2) = 1 ± 2n = 1025
 2n = 1024 = 210
 n = 10
 f(x) = 1 + x10.
 f'(x) = 10x9.

 lim f ( x )  lim 10  x 9  10  29  5  210


x 2 x 2

SECTION - B
Objective Type Questions (More than one options are correct)

b
1. lim (1  ax ) x  e 2 , where a, b N, such that a + b = 3, then the value of (a, b) is
x 0

(1) (16, 8) (2) (8, 4) (3) (2, 1) (4) (1, 2)


Sol. Answer (3, 4)
We have

b
lim (1  ax ) x  e 2 and a + b = 3
x 0

Aakash Educational Services Pvt. Ltd. Regd. Office : Aakash Tower, 8, Pusa Road, New Delhi-110005 Ph.011-47623456
184 Continuity and Differentiability Solutions of Assignment (Set-2)

lim ab
 e x 0  e2  eab  e 2
 ab = 2 …(1)
a+b=3 …(2)
Solving (1) and (2) we get
a = 2 or 1
b = 1 or 2
Hence (a, b) = (2, 1) or (1, 2)

⎛ 2 2 ⎞⎟
2. lim sec 1⎜  exists, then  belongs to
x 1 ⎜ ln x x  1 ⎟
⎝ ⎠

(1) (, 2 ] (2) [ 2, ) (3) ( ,  2]  [ 2,  ) (4) None of these

Sol. Answer (1, 2, 3)


In order that

⎛ 2 2 ⎞
lim sec 1 ⎜ 
x 1 ⎜ n x x  1 ⎟⎟ exists, we must have
⎝ ⎠

⎛ 2 2 ⎞
lim ⎜  ⎟ 1
x 1 ⎜ n x x  1 ⎟⎠

⎛ 1 1 ⎞
 lim  2 ⎜  ⎟ 1
x 1 ⎝ n x x  1 ⎠

⎛ x  1  n x ⎞
 lim  2 ⎜ ⎟ 1
x 1 ⎝ n x ( x  1) ⎠

2
 1
2

2
 1
2
 2 – 2  0

   2 or  2    (– , – 2 ]  [ 2 , )

1
3. Given the function f ( x )  . The points of discontinuity of the composite function f [f {f(x)}] are given by
1 x
(1) 0 (2) 1 (3) 2 (4) –1

Aakash Educational Services Pvt. Ltd. Regd. Office : Aakash Tower, 8, Pusa Road, New Delhi-110005 Ph.011-47623456
Solutions of Assignment (Set-2) Continuity and Differentiability 185

Sol. Answer (1, 2)

1
f(x) =
1 x
f(x) discontinuous at x = 1

1 1 x 1 x 1
f(f(x)) = = = 1 
1 x x x
1
1 x

f(f(x)) discontinuous at x = 0

1 x
f(f(f(x))) =  x
x 1 x  x 1
1
x

Although f(f(f(x))) = x but f(f(f(x))) exist only when f(f(x)) and f(x) exist.
Hence f(f(f(x))) is discontinuous at x = 0 and x = 1
4. A function is defined as follows

⎧x 3 ; x 2  1

f (x)  ⎨ then function is
⎪⎩ x ; x 2  1

(1) Continuous at x = 1 (2) Differentiable at x = 1


(3) Continuous but not differentiable at x = 1 (4) None of these
Sol. Answer (1, 3)
We have,
1
⎧⎪ x 3 , x 2  1
f(x) = ⎨ –2 –1
⎪⎩ x, x 2  1
0 1
⎪⎧ x 3 ,  1  x  1 –1
= ⎨
⎪⎩ x, x  1 or x  1

From graph it is clear that f(x) is continuous at x = 1 but not


differentiable at x = 1.

( x  1)
5. Let f ( x )  2
then
(2 x  7 x  5)

1 1
(1) lim f ( x )   (2) lim f ( x )  
x 1 3 x 0 5

(3) lim f ( x )  0 (4) lim f ( x ) does not exist


x  5
x
2

Aakash Educational Services Pvt. Ltd. Regd. Office : Aakash Tower, 8, Pusa Road, New Delhi-110005 Ph.011-47623456
186 Continuity and Differentiability Solutions of Assignment (Set-2)

Sol. Answer (1, 2, 3, 4)

x 1
f (x) 
2
2x  7 x  5

x 1 x 1
 f (x)  =
2
2x  2x  5 x  5 2 x( x  1)  5( x  1)

1
 f (x) 
2x  5

1
 lim f ( x )  
x 1 3

1
Also, lim f ( x )  
x 0 5

1 1
and lim f ( x )   =  lim f ( x )  0
x  2( )  5  x 

lim f(x) 

Hence lim f ( x ) does not exist at x = 5/2.


x 

dy
6. If y  tan x tan 2 x tan 3 x, then equals
dx
(1) 3sec23x tanx tan2x + 2sec22x tan3x tanx + sec2x tan2x tan3x
(2) 2y(cosec2x + 2 cosec 4x + 3 cosec 6x)
(3) 3sec23x – 2sec22x – sec2x
(4) sec2x + 2sec22x + 3sec23x
Sol. Answer (1, 2, 3)
y = tanx tan2x tan3x
Differentiating,
dy d tan3 x d tan x d tan 2 x
 tan x tan 2 x  tan 2 x tan3 x  tan x tan3 x
dx dx dx dx

= tan x tan 2 x sec 2 3 x.3  tan 2 x tan3 x sec 2 x  tan x tan3 x sec 2 2 x 2
dy
dx
= tan 2x tan 3x sec2x + 2 tanx tan 3x sec2 2x + 3 tanx tan 2x sec23x
Again, y = tanx tan2x tan3x
y = tan 3x – tan 2x – tan x
Differentiating,

dy
 sec 2 3 x.3  sec 2 2 x.2  sec 2 x = 3sec23x - 2sec22x - sec2x
dx

Aakash Educational Services Pvt. Ltd. Regd. Office : Aakash Tower, 8, Pusa Road, New Delhi-110005 Ph.011-47623456
Solutions of Assignment (Set-2) Continuity and Differentiability 187

Again, y = tan x tan 2x tan 3x


 log y = log tan x + log tan 2x + log tan 3x

dy
  2y cosec 2x  2cosec 4x  3cosec 6x 
dx
7. Which of the following function(s) not defined at x = 0 has / have removable discontinuity at x = 0?

1 ⎛ | sin x | ⎞  1
(1) (2) cos⎜ ⎟ (3) x sin (4)
(1  2 cot x
) ⎝ x ⎠ x ln | x |

Sol. Answer (2, 3, 4)

| sin x | | sin x |
We observe that  1 for x  0 + and  –1, for x  0–
x x

⎛ | sin x | ⎞
But in either case lim cos ⎜ ⎟  cos1
x 1 ⎝ x ⎠

 1
Also, xsin  0 as x  0 and  0 as x  0
x ln | x |

| sin x |  1 1
Hence the functions , xsin , ln | x | have removable discontinuity while f(x) = has non-
x x 1  2cot x
removable discontinuity as L.H.L.  R.H.L.
8. Let f : [a, b]  [1, ) be a continuous function and let g : »  » be defined as


⎪ 0 if x  a,
⎪ x
g ( x )  ⎨∫ f (t ) dt if a  x  b,
a
⎪ . Then [JEE(Advanced)-2014
⎪ b f (t )dt
⎩ ∫a
if x  b

(1) g(x) is continuous but not differentiable at a


(2) g(x) is differentiable on »
(3) g(x) is continuous but not differentiable at b
(4) g(x) is continuous and differentiable at either a or b but not both
Sol. Answer (1, 3)
a ah
g(a–) = 0, g(a) = ∫ f (t )dt  0 , g(a ) = lim ∫
a
+
h 0 a
f (t )dt  0

b h b
g(b–) = lim
h 0 a ∫ f (t )dt  ∫
a
f (t )dt

b
g(b) = ∫a
f (t )dt  g (b  )

Aakash Educational Services Pvt. Ltd. Regd. Office : Aakash Tower, 8, Pusa Road, New Delhi-110005 Ph.011-47623456
188 Continuity and Differentiability Solutions of Assignment (Set-2)

Hence g(x) is continuous at x = a as well as x = b

g (a  h )  g (a )
Now, g’(a–) = lim 0
h 0 h

ah

g’(a+) = lim
g (a  h )  g (a )
 lim

a
f (t )dt  0
 lim
f (a  h )
= f(a) ( 0)
h 0 h h 0 h h 0 1
Hence g(x) is not differentiable at x = a.

b h b

g’(b–) = lim
g (b  h)  g (b)
 lim

a
f (t )dt  ∫
a
f (t )dt
 lim
f (b  h)
 f (b )(  0)
h 0 h h 0 h h 0 1

b h b

g’(b+) = lim
g (b  h)  g (b)
 lim
∫ a
f (t )dt  ∫ a
f (t )dt
 lim
f (b  h)
 f (b )(  0)
h 0 h h 0 h h 0 1
Hence g(x) is not differentiable at x = b.
9. Consider the hyperbola H : x2 – y2 = 1 and a circle S with center N(x2, 0). Suppose that H and S touch each
other at a point P(x1, y1) with x1 > 1 and y1 > 0. The common tangent to H and S at P intersects the x-axis
at point M. If (l, m) is the centroid of the triangle PMN, then the correct expression(s) is(are)
[JEE(Advanced)-2015]

dl 1 dm x1
 1  2 for x1  1  for x1  1
 
(1) (2)
dx1 3 x1 dx1 3 x12  1

dl 1 dm 1
(3) dx  1  3 x 2 for x1  1 (4) dy  3 for y1  0
1 1 1

Sol. Answer (1, 2, 4)


Equation of tangent at P on hyperbola.
xx1 – yy1 = 1

⎛ 1 ⎞ P (x1, y1)
Point M ⎜ , 0⎟
⎝ x1 ⎠
N
Equation of normal at P (x2, 0)
(–1, 0) M (1, 0)

x y
 2
x1 y1

since (x2, 0) satisfies its


x2 = 2x1

⎛ 1 y1 ⎞
Centroid  l , m   ⎜ x1  ,
⎝ 3 x1 3 ⎟⎠

Aakash Educational Services Pvt. Ltd. Regd. Office : Aakash Tower, 8, Pusa Road, New Delhi-110005 Ph.011-47623456
Solutions of Assignment (Set-2) Continuity and Differentiability 189

dl 1
 1 2
dx1 3 x1

dm 1

dy1 3

x12  y 12  1

y1  x12  1

x12  1
m
3

dm x1

dx1 3 x12  1

Alternetive Method

3 sec   cos 
l 
3
y
tan  y1 P (sec, tan)  (x1y1)
m 
3 3

dl 3 sec  tan  – sin  1 1 (l, m)


  1– 2
 1– 2 N
dx1 3 sec  tan  3 sec  3 x1
(cos, 0) (2sec, 0)  (x2, 0)
2
sec 
dm 3 cosec  x1
  
dx1 sec  tan  3 3 x12 – 1

dm 1

dy1 3

⎧ x
⎪ g ( x ), x  0
10. Let g : R  R be a differentiable function with g(0) = 0, g(0) = 0 and g(1)  0. Let f ( x )  ⎨| x |
⎪ 0, x 0

and h(x) = e|x| for all x  R. Let (foh)(x) denote f(h(x)) and (hof)(x) denote h(f(x)). Then which of the following
is (are) true? [JEE(Advanced)-2015]
(1) f is differentiable at x = 0 (2) h is differentiable at x = 0
(3) foh is differentiable at x = 0 (4) hof is differentiable at x = 0
Sol. Answer (1, 4)

⎧g ( x ) x 0

f ( x )  ⎨0 x 0
⎪–g ( x ) x  0

Aakash Educational Services Pvt. Ltd. Regd. Office : Aakash Tower, 8, Pusa Road, New Delhi-110005 Ph.011-47623456
190 Continuity and Differentiability Solutions of Assignment (Set-2)

LHD : lim– (– g ( x ))  – lim– g ( x )  –0  0


x 0 x 0


RHD : lim g ( x )  g (0 )  0
x 0

Hence f(x) is differentiable at x = 0

⎧⎪ e x x0
h( x )  e|x|  ⎨
–x
⎪⎩e x0

LHD : lim– (– e – x )  –1
x 0

x
RHD : lim (e )  1
x 0

Hence h(x) is not differentiable at x = 0

⎧⎪ e|x|
f (h( x ))  ⎨ |x| g (e|x| )
⎪⎩| e |

x
⎪⎧ g (e ) x  
f (h( x ))  ⎨
–x
⎪⎩g (e ) x  0

–x –x
LHD : lim– g (e )  – e  g (1)  –1  – g (1)
x 0

x x
RHD : lim g (e )  e  g (1)
x 0

Hence foh is non-differentiable at x = 0

⎧ 1 x 0
⎪ ⎧⎪ 1 x0
h(f ( x ))  ⎨ x g ( x )  ⎨ |g ( x )|
⎪e |x| x  0 ⎪⎩e x0


LHD : lim– e|g ( x )|  g ( x )  e|g (0 )|
 (g (0 – ))  0
x 0


RHD : lim e|g ( x )|   g ( x )  e|g (0 )|
  g (0  )  0

x 0

11. Let f :   , g :    and h :    be differentiable functions such that f(x) = x3 + 3x + 2,


g(f(x)) = x and h(g(g(x))) = x for all x  . Then [JEE(Advanced)-2016]

1
(1) g '(2)  (2) h'(1) = 666 (3) h(0) = 16 (4) h(g(3)) = 36
15
Sol. Answer (2, 3)

g (f ( x ))  f ( x )  1

g (2)  f (0)  1

Aakash Educational Services Pvt. Ltd. Regd. Office : Aakash Tower, 8, Pusa Road, New Delhi-110005 Ph.011-47623456
Solutions of Assignment (Set-2) Continuity and Differentiability 191

1
g (2) 
f (0)

f ( x )  3 x 2  3

1
g (2) 
3
h(g(g(x)) = x
h(g(g(f(x))) = f(x)
h(g(x)) = f(x)
h(g(3)) = f(3) = 38
h(g(f(x))) = f(f(x))
h(x) = f(f(x))

h( x )  f (f ( x ))  f ( x )

h(1)  f (f (1))  f (1) = 111 × 6 = 666


h(0) = f(f(0)) = f(2) = 16
12. Let a, b   and f :    be defined by f(x) = acos(|x3 – x|) + b|x|sin(|x3 + x|). Then f is
[JEE(Advanced)-2016]
(1) Differentiable at x = 0 if a = 0 and b = 1 (2) Differentiable at x = 1 if a = 1 and b = 0
(3) NOT differentiable at x = 0 if a = 1 and b = 0 (4) NOT differentiable at x = 1 if a = 1 and b = 1
Sol. Answer (1, 2)

f ( x )  a cos(| x 3  x |)  b | x | sin(| x 3  x |)

⇒ f ( x )  a cos( x 3  x )  bx sin( x 3  x ) for x  R

So f(x) is differentiable for all x  R

⎡ 1 ⎤ ⎡ 1 ⎤ 2
13. Let f : ⎢  , 2⎥  » and g : ⎢  , 2⎥  » be functions defined by f ( x )  ⎡⎣ x  3 ⎤⎦ and
⎣ 2 ⎦ ⎣ 2 ⎦
g ( x )  | x | f ( x )  | 4 x  7 | f ( x ) , where [y] denotes the greatest integer less than or equal to y for y  » . Then
[JEE(Advanced)-2016]

⎡ 1 ⎤
(1) f is discontinuous exactly at three points in ⎢  , 2⎥
⎣ 2 ⎦

⎡ 1 ⎤
(2) f is discontinuous exactly at four points in ⎢  , 2⎥
⎣ 2 ⎦

⎛ 1 ⎞
(3) g is NOT differentiable exactly at four points in ⎜  , 2 ⎟
⎝ 2 ⎠

⎛ 1 ⎞
(4) g is NOT differentiable exactly at five points in ⎜  , 2 ⎟
⎝ 2 ⎠

Aakash Educational Services Pvt. Ltd. Regd. Office : Aakash Tower, 8, Pusa Road, New Delhi-110005 Ph.011-47623456
192 Continuity and Differentiability Solutions of Assignment (Set-2)

Sol. Answer (2, 4)

Given, f ( x )  ⎡ x 2  3 ⎤
⎣ ⎦

g ( x )  | x | f ( x ) | 4 x  7 | f ( x )
= | x | [x2 – 3] + | 4x – 7 | [x2 – 3]
= | x | [x2] – 3 | x | + | 4x – 7 | [x2] – 3 | 4x – 7 |

7
Discontinuous at x = 1, 2, 3,2 not differentiable 0,1, , 2, 3
4

14. Let f : »  (0, 1) be a continuous function. Then, which of the following function(s) has(have) the value zero
at some point in the interval (0, 1)? [JEE(Advanced)-2017]

x
(1) x  ∫ 2 f (t )cos t dt (2) x9 – f(x)
0


x
(3) e x  ∫ f (t )sin t dt (4) f ( x )  ∫ 2 f (t )sin t dt
0 0

Sol. Answer (1, 2)



x
g ( x )  x  ∫ 2 f (t )cos t dt
0


g (0)  0  ∫ 2 f (t )cos t dt  0 as 0  f (t ) cos t < 1
0


1
g (1)  1  ∫ 2
f (t )cos t dt  0
0

g ( x )  x 9  f ( x )  g (0)  0  f (0)  0

g (1)  1  f (1)  0
Also for (0, 1)
x
e x  ∫ f (t )sin t dt  0
0

 /2
Also, f ( x )  ∫ f (t )sin t dt  0 x  (0,1)
0

SECTION - C
Linked Comprehension Type Questions
Comprehension I
A square is inscribed in a circle of radius R, a circle is inscribed in the square, a new square in the circle and so on for
n times.
1. Sum of the areas of all cirmcles is

⎛ ⎛ 1 ⎞n ⎞ ⎛ ⎛ 1 ⎞n ⎞ ⎛ ⎛ 1 ⎞n ⎞ ⎛ ⎛ 1 ⎞n ⎞
(1) 4R 2 ⎜1  ⎜ ⎟ ⎟ (2) 2R 2 ⎜1  ⎜ ⎟ ⎟ (3) 3R 2 ⎜1  ⎜ ⎟ ⎟ (4) R 2 ⎜1  ⎜ ⎟ ⎟
⎜ ⎝2⎠ ⎟ ⎜ ⎝2⎠ ⎟ ⎜ ⎝3⎠ ⎟ ⎜ ⎝2⎠ ⎟
⎝ ⎠ ⎝ ⎠ ⎝ ⎠ ⎝ ⎠

Aakash Educational Services Pvt. Ltd. Regd. Office : Aakash Tower, 8, Pusa Road, New Delhi-110005 Ph.011-47623456
Solutions of Assignment (Set-2) Continuity and Differentiability 193

Sol. Answer (2)

R R
Radius of circles are R, , ,. . .
2 2 R
 The sum of the areas of the circles

R 2 R 2
= A = R2 +   . . . to n terms
2 4

2⎡ 1 1 1 ⎤
= R ⎢1   2  . . .  n 1 ⎥
⎣ 2 2 2 ⎦

⎡ ⎛ 1 ⎞n ⎤
⎢1  ⎜ ⎟ ⎥ n
R 2⎢ ⎝ 2 ⎠ ⎥  2R 2 ⎡⎢1  ⎛ 1 ⎞ ⎤⎥
= ⎢ 1 ⎥ ⎜ ⎟
⎢⎣ ⎝ 2 ⎠ ⎥⎦
⎢ 1 ⎥
⎢⎣ 2 ⎥

2. The limit of sum of areas of all the squares as n  is
(1) 2R 2 (2) 3R 2 (3) 4R 2 (4) 8R 2
Sol. Answer (3)

R R
Side of squares are 2R, R, , ,. . .
2 2

R2 2R 2
Sum of areas of the squares = A = 2 R2 + R2 + +...+ = 1 = 4R
2
2 1
2
3. The limit of sum of areas of all the circles as n 
(1) 2R 2 (2) 3R 2 (3) 4R 2 (4) 8R 2
Sol. Answer (1)

⎡ 1⎤
Sum of areas of n circles = 2R2 ⎢1  n ⎥
⎣ 2 ⎦

⎡ 1⎤
lim 2R 2 ⎢1  ⎥ = 2R2 [1 – 0]
n  ⎣ 2n ⎦
 The limit of sum of areas of all the circles as n  = 2R2

Comprehension II
A function f : R  R satisfies the following conditions

(i) f ( x )  0 x  R

(ii) f ( x  y )  f ( x )f ( y )  x, y  R
(iii) f(x) is differentiable
(iv) f (0) = 2

Aakash Educational Services Pvt. Ltd. Regd. Office : Aakash Tower, 8, Pusa Road, New Delhi-110005 Ph.011-47623456
194 Continuity and Differentiability Solutions of Assignment (Set-2)

1. The derivative of f(x) satisfies the equation

(1) f ( x  y )  f ( x )  f ( y ) (2) f ( x  y )  f ( x )  f ( y )

(3) f ( x  y )  f ( x )f ( y ) (4) f ( x  y )  f ( x )  f ( y )
2. The value of f(0) is

1
(1) 1 (2) –1 (3) 2 (4)
2

f ( x )  f ( x )
3. lim
x 0 x
(1) 1 (2) 2 (3) 3 (4) 4
Solutions of Comprehension II
f : R  R, f(x)  0, x  R
f(x + y) = f(x) f(y) x, y  R
f(x) is differentiable
f'(0) = 2
1. Answer (3)

f ( x  h)  f ( x )
f'(x) = lim
h 0 h

f ( x ).f (h )  f ( x )
= lim
h 0 h

⎡ f (h )  1⎤
= lim f ( x ) ⎢ ⎥
h 0 ⎣ h ⎦

f (0  h )  f (0)
= f ( x ) lim
h 0 h
= f(x).f'(0) = f'(x) = 2f(x), f'(y) = 2f(y)

 f '( x  y )  2f ( x  y )

f '( x )
 f '( x  y )  2f ( x ).f ( y ) = 2f ( y )
2

 f '( x  y )  f '( x ).f ( y )


2. Answer (1)

f ( x  y )  f ( x ).f ( y )
Putting x = 0, y = 0 we get

f (0)  [f (0)]2
 f(0) [f(0) – 1] = 0
 f(0) = 1 as f(0)  0

Aakash Educational Services Pvt. Ltd. Regd. Office : Aakash Tower, 8, Pusa Road, New Delhi-110005 Ph.011-47623456
Solutions of Assignment (Set-2) Continuity and Differentiability 195

3. Answer (4)

f ( x )
f '( x )  2f ( x )  =2
f (x)
 n f(x) = 2x + c, on integration, where c is a constant of integration
Putting x = 0, we get
n f(0) = 2 × 0 + c c=0
 n f(x) = 2x  f(x) = e2x

f ( x )  f ( x ) e2 x  e 2 x
Now, lim = lim
x 0 x x 0 x

Applying L. Hospital rule = lim 2e2 x  2e 2 x = 4


x 0

Comprehension III
The notion of limit of a single variable function can be extended to the function of two variables. When
(x, y)  (a, b) i.e., x  a, y  b simultaneously, then lim f ( x, y ) can be evaluated by the path along with (x, y)
( x, y )( a, b )

approaches to (a, b).

x2 x2 x2 1
For example lim where (x, y)  (0, 0) along the curve y 2 = x 3 can be evaluated as lim  lim 3 / 2  lim x 2  0
x 0 y x 0 y x 0 x x 0
y 0 y 0
3/2
y x

Then,

x3
1. lim . where ( x, y)  (0, 0) along the curve y 2 = x 5 equals
x 0 y
y 0

(1) 0 (2) 1 (3) 2 (4) 3


Sol. Answer (1)

x3 x3
lim  lim 0
x 0 y x 0 x 5/2
y 0

x 2  sin y
2. lim when (x, y)  (0, 0) along the curve y = x 2 is equal to
x 0 y 2  sin x 2
y 0

(1) 0 (2) 1 (3) 2 (4) 3


Sol. Answer (3)

⎛ sin x 2 ⎞ 2
x2  ⎜ ⎟x
x 2  sin y x 2  sin x 2 ⎜ x2 ⎟ x2  x2 2
lim  lim  lim ⎝ ⎠  lim  lim 2
2
x 0 y  sin x 2 4
x 0 x  sin x 2 x 0 2 4
⎛ sin x ⎞ 2 x 0 x  x 2 2
x 0 x  1
4
y 0 x ⎜ ⎟x
⎜ x2 ⎟
⎝ ⎠

Aakash Educational Services Pvt. Ltd. Regd. Office : Aakash Tower, 8, Pusa Road, New Delhi-110005 Ph.011-47623456
196 Continuity and Differentiability Solutions of Assignment (Set-2)

y3
3. lim , when (x, y)  (1, 0) along the line y = x –1 is equal to
x 1 x3  y 2 1
y 0

(1) 0 (2) 1 (3) 2 (4) 3


Sol. Answer (1)

y3 ( x  1)3 3( x  1)2
lim  lim  lim 0
x 1 x3  y 2  1 x 1 x 3  ( x  1)2  1 x 1 3 x 2  2( x  1)
y 0

Comprehension IV
We have,

x x x x x sin x
f ( x )  cos cos 2 cos 3 cos 4 ....cos n 
2 2 2 2 2 x
2n sin n
2
using the identity

1. lim lim f ( x ) equals


n   x 0

1
(1) 0 (2) 1 (3) 2 (4)
2
n
1 ⎛ x ⎞
2. lim
n 
∑2
k 1
k
tan ⎜ k ⎟ equals
⎝2 ⎠

1 1
(1) x  tan x (2)  cot x (3) x  cot x (4)  tan x
x x
n
1 ⎛ x ⎞
3. lim
n 
∑2
k 1
2k
sec 2 ⎜ k ⎟ equals
⎝2 ⎠

1 1 1
(1) cosec 2 x  (2) cosec 2 x  (3) sin2 x  x 2 (4) sin2 x 
x2 x2 x2
Solutions of Comprehension IV

x x x ⎛ x ⎞ sin x
cos .cos .cos ....cos ⎜
2 3 ⎟  f (x)
2 2 2 ⎝ 2n ⎠ 2n sin ⎛ x ⎞
⎜ n⎟
⎝2 ⎠
1. Answer (2)
sin x (sin x ) sin x
lim lim  lim  lim 1
n  x 0 n ⎛ x ⎞ n  ⎛ ⎛ x ⎞⎞ x 0 x
2 sin ⎜ ⎟ ⎜ sin ⎜ n ⎟⎟
⎝ 2n ⎠ ⎝2 ⎠ ⎟.⎛ x ⎞
2n. ⎜
⎜ ⎛ x ⎞ ⎟ ⎜⎝ 2n ⎟⎠
⎜ ⎜ n⎟ ⎟
⎝ ⎝2 ⎠ ⎠

Aakash Educational Services Pvt. Ltd. Regd. Office : Aakash Tower, 8, Pusa Road, New Delhi-110005 Ph.011-47623456
Solutions of Assignment (Set-2) Continuity and Differentiability 197

2. Answer (2)

x x x x sin x
cos .cos .cos ...cos 
2 3
2 2 2 2n ⎛ x ⎞
2n sin ⎜ ⎟
⎝ 2n ⎠

⎛ x⎞ ⎛ x ⎞ ⎛ ⎛ x ⎞⎞
log ⎜ cos ⎟  log ⎜ cos ⎟  ....  log(sin x )  log ⎜ 2n.sin ⎜ ⎟ ⎟ differentiating the whole equation.
2
⎝ 2⎠ ⎝ 2 ⎠ ⎝ ⎝ 2n ⎠ ⎠

1 ⎛x⎞ 1 ⎛ x ⎞ 1 ⎛ x ⎞
 .tan ⎜ ⎟  tan ⎜ ⎟  ........  cot x  n cot ⎜ n ⎟
2 ⎝2⎠ 2 2 2
⎝2 ⎠ 2 ⎝2 ⎠

1 x 1 ⎛ x ⎞ 1 ⎛ x ⎞
 tan  tan ⎜  .....  cot ⎜ ⎟  cot x
2 22 2⎟ n …(i)
2 ⎝2 ⎠ 2 ⎝ 2n ⎠

n
1 ⎛ x ⎞ 1 ⎛ x ⎞
 ∑ 2k tan ⎜⎝ 2k ⎟⎠  2n cot ⎜⎝ 2n ⎟⎠  cot x
k 1

3. Answer (1)
Again differentiating equation (i) of Q. 2, we get

1 x 1 ⎛ x ⎞ 1 ⎛ x ⎞
sec 2  sec 2 ⎜  .......   cosec 2 ⎜ 2
⎟  cosec x
2 2 24 2⎟ 2n
2 ⎝2 ⎠ 2 ⎝ 2n ⎠

Comprehension V
The greatest integer function [x] is defined as the greatest integer  x and {x} is defined as {x} = x – [x].

a2[ x]{ x} – 1
1. lim is equal to
x0 2[ x]  {x}

1 1
(1) lna (2) 1–a (3) 1 (4) 1
a a

Sol. Answer (1)

a2[ x ] { x }  1 a 2[0  h ] {h }  1 ah  1
lim  lim  lim  log a
x 0 2[ x ]  { x } h 0 2[0  h]  {h} h 0 h

a 2[ x ] { x }  1
2. lim is equal to
x 0  2[ x ]  { x }

1 1
(1) Ina (2) 1–a (3) 1 (4) 1
a a

Aakash Educational Services Pvt. Ltd. Regd. Office : Aakash Tower, 8, Pusa Road, New Delhi-110005 Ph.011-47623456
198 Continuity and Differentiability Solutions of Assignment (Set-2)

Sol. Answer (3)

a 2[ x ] { x }  1
lim
x 0 2[ x ]  { x }

a2[0 h ] {0 h }  1 a 1  1 1
= lim  lim   1
h 0 2[0  h]  {0  h} h 0 1 a

3. Let f(x) = [x ] 2 – [x 2], then

(1) lim f ( x )  0 (2) lim f ( x )  0


x 2 x 1

(3) lim f ( x )  1 (4) lim f ( x ) does not exist


x 0 x1

Sol. Answer (2)

lim [ x ]2  [ x 2 ]
x 1

Here we may generalize the result at x = k (a integer)

R.H.L. at (x = k) = lim [k  h]2  [(k  h )2 ]  k 2  k 2  0


h 0

L.H.L. at ( x  k )  lim [k  h]2  [(k  h )2 ] = (k – 1)2 – (k2 – 1) = 2k – 2.


h 0

If R.H.L. = L.H.L
 2k – 2 = 0 k = 1

Hence lim f ( x ) exist and lim f ( x )  0


x 1 x 1

Comprehension VI
Let f(x) = min{x – [x], –x – [–x]}, –2  x  2
g(x) = | 2 – | x – 2||, – 2  x  2

| sin x |
and h( x )  ,  2  x  2 and x  0. be three given functions where [x] denotes the greatest integer  x. Then
sin x

1. The number of solution(s) of the equation x 2 + (f(x ))2 = 1 (–1  x  1) is/are


(1) 0 (2) 2 (3) 4 (4) 6
2. The range of f(x ) is

⎡ 1⎤ ⎡ 1 ⎤
(1) ⎢0, ⎥ (2) [0, 1] (3) [0, 2] (4) ⎢ 2 , 0⎥
⎣ 2⎦ ⎣ ⎦
3. The sum of all the roots of the equation g(x)–h(x) = 0 is
(1) 3 (2) 1 (3) –2 (4) 7

Aakash Educational Services Pvt. Ltd. Regd. Office : Aakash Tower, 8, Pusa Road, New Delhi-110005 Ph.011-47623456
Solutions of Assignment (Set-2) Continuity and Differentiability 199

Solutions of Comprehension VI

f ( x )  min{{ x }, { x }},  2  x  2

g ( x )  | 2 | x  2 ||,  2  x  2

= | 2  (2  x ) |  | x |

| sin x |
h( x )  ,2 x  2
sin x

⎧ 1 if 0x2
h( x )  ⎨
⎩ 1 if 2  x  0

1. Answer (2)
y
2
2 2 y=1–x
x  (f ( x ))  1
⎛ 1⎞
⎜0, 4⎟ 2
(f ( x ))2  1  x 2 ⎝ ⎠
y = (f(x))
x
To find the number of selection we draw two graphs –2 3 –1 1 1 1 3 2
– –
2 2 2 2
y  (f ( x ))2 and y  1  x 2

(i) f ( x )  min{{ x }, { x }}

2
The graph of y  (f ( x )) and y  1  x 2 can be shown as
Hence only two solutions x = ± 1 exists.
2. Answer (1) y
1
The graph of y = f(x) is given by y=
2

⎡ 1⎤ x
Clearly the range of y = (f(x) is ⎢0, ⎥ . –2 –1 0 1 2
⎣ 2⎦
3. Answer (2)

Comprehension VII

Let lim f ( x ) be a finite number, where


x0

sin x  ae x  be  x  c ln(1  x )
f (x)  , a, b, c  R
x3
1. The value of a is

1 1
(1)  (2) 0 (3) (4) 1
2 2
2. The value of b is

1 1
(1)  (2) (3) 0 (4) 1
2 2

Aakash Educational Services Pvt. Ltd. Regd. Office : Aakash Tower, 8, Pusa Road, New Delhi-110005 Ph.011-47623456
200 Continuity and Differentiability Solutions of Assignment (Set-2)

3. The value of c is

1 1
(1)  (2) (3) 0 (4) 2
2 2
Solutions of Comprehension VII
1. Answer (1)
2. Answer (2)
3. Answer (3)

sin x  ae x  be  x  c ln(1  x )
lim f ( x )  lim
x 0 x 0 x3

⎛ x3 x5 ⎞ ⎛ x x2 ⎞ ⎛ x x2 ⎞ ⎛ x2 x3 ⎞
⎜x   ... ⎟  a ⎜ 1    ... ⎟  b ⎜ 1   ... ⎟  c ⎜ x    ... ⎟
⎜ 3! 5! ⎠ ⎟ ⎜ ⎟ ⎜ ⎟ ⎜ ⎟
⎝ ⎝ 1! 2! ⎠ ⎝ 1! 2! ⎠ ⎝ 2 3 ⎠
= lim
x 0 x3

⎛a b c⎞ ⎛ 1 a b⎞ ⎛ a b c⎞
(a  b )  x (1  a  b )  x 2 ⎜   ⎟  x 3 ⎜    ⎟  x 4 ⎜   ⎟  ...
⎝2 2 2⎠ ⎝ 6 6 6⎠ ⎝ 24 24 4 ⎠
= lim
x 0 x3

If lim f ( x ) infinite then


x 0

a+b=0 …(i)
1+a–b=0 …(ii)

a b c
  0 …(iii)
2 2 2
By (i), (ii) and (iii)

1
a= 
2

1
b =
2
c=0

SECTION - D
Assertion-Reason Type Questions
1  cos(1  cos x )
1. STATEMENT-1 : f ( x )  is continuous if f (0)  1 .
x4 8
and

1
STATEMENT-2 : lim f ( x )  lim f ( x )  .

x 0
x 0 8

Aakash Educational Services Pvt. Ltd. Regd. Office : Aakash Tower, 8, Pusa Road, New Delhi-110005 Ph.011-47623456
Solutions of Assignment (Set-2) Continuity and Differentiability 201

Sol. Answer (1)

1  cos(1  cos x )
Statement–1 : f (0)  lim
x 0 x4

2 4
⎛ x⎞ ⎛ x⎞ ⎡ ⎛ 2 x ⎞⎤ ⎛⎛ x⎞⎞
1  cos ⎜ 2sin2 ⎟ 2sin2 ⎜ sin2 ⎟ sin sin ⎜ ⎜ sin ⎟ ⎟
⎝ 2⎠ ⎝ 2⎠ 2 ⎢ ⎜⎝ 2 ⎟⎠ ⎥⎥ ⎝ 2 ⎠⎟  1
= lim  lim  lim ⎢ .⎜
x 0 x4 x 0 x4 x 0 16 ⎢ ⎛ 2 x⎞ ⎥ ⎜ x ⎟ 8
⎢ ⎜⎝ sin 2 ⎟⎠ ⎥ ⎜ 2 ⎟
⎣ ⎦ ⎝ ⎠

 Statement–1 is true.
Statement–2 : LHL = RHL.
Statement 1 is true Statement 2 is true, Statement–2 is correct explanation of Statement–1.

1
2. STATEMENT-1 : If f ( x )  log x 2 (log x ), then f (e )  .
e
and

log b
STATEMENT-2 : If a > 0, b > 0 and a  b then loga b  .
log a

Sol. Answer (4)

log(log x )
Statement–1 : f ( x )  logx 2 (log x ) = 2log x

1 log(log x )

1x x
f '( x ) 
2 (log x )2

1
f '(e ) 
2e
Statement–1 is false.
Statement–2 : is true.
3. STATEMENT-1 : Let f be a twice differentiable function such that f (x) = g(x) and f (x) = –f(x). If h(x) = [f(x)]2
+ [g(x)]2, h(1) = 8 and h(0) = 2  h(2) = 14.
and
STATEMENT-2 : h(x) = 0.
Sol. Answer (2)
Statement–1 : f'(x) = g(x)  f''(x) = g'(x)
f''(x) = –f(x)
Also h'(x) = [f(x)]2 + [g(x)]2

 h ''( x )  2f ( x )f '( x )  2g ( x )g '( x ) = 2f ( x )g ( x )  2g ( x )f ''( x ) = 2f ( x )g ( x )  2g ( x )f ( x ) = 0

Aakash Educational Services Pvt. Ltd. Regd. Office : Aakash Tower, 8, Pusa Road, New Delhi-110005 Ph.011-47623456
202 Continuity and Differentiability Solutions of Assignment (Set-2)

 h'(x) = a, a is a constant of integration  h(x) = ax + b h(1) = a + b = 8

 h(0)  b  2  a = 6.  h( x )  6 x  2  h(2) = 14
i.e., Statement–1 is true.
Statement–2 : h''(x) is true. Statement-2 is not a correct explanation of statement-1

3 3
⎧⎪ ⎛ dy ⎞ 2 ⎫⎪ 2 ⎧⎪ ⎛ dx ⎞ 2 ⎫⎪ 2
⎨1  ⎜ ⎟ ⎬ ⎨1  ⎜⎜ ⎟⎟ ⎬
⎪⎩ ⎝ dx ⎠ ⎪⎭ ⎪⎩ ⎝ dy ⎠ ⎪⎭
4. STATEMENT-1 : For the function y  f ( x ),  , where f(x) is twice differentiable
d 2y d 2x
dx 2 dy 2
function.
and

1
dy dx d 2y d ⎛ dy ⎞
STATEMENT-2 :  and  .
dx dy dx 2 dx ⎜⎝ dx ⎟⎠

Sol. Answer (1)

Statement–1 : y  f ( x )
Differentiating it w.r.t. x Differentiating again w.r.t. y, we get

dy dx
 f '( x ) 1  f '( x )
dx dy

d 2y dx 1
 f ''( x ) 
dx 2 dy f '( x )

d 2x 1 dx f ''( x )
 .f ''( x ). = 
dy 2
f '( x )2 dy {f '( x )}3

3/2
⎧⎪ ⎛ dy ⎞2 ⎫⎪
⎨1  ⎜
1  (f '( x ))2 
⎟ ⎬ 3/2
⎪ ⎝ dx ⎠ ⎪⎭
Now, ⎩  …(i)
d 2y f ''( x )
dx 2

3/2
⎧⎪ ⎛ dx ⎞2 ⎫⎪ 3/2
⎪⎧ 1 ⎪⎫
⎨1  ⎜ ⎟ ⎬ ⎨1  2⎬ {1  (f '( x ))2 }3/2 .(f '( x ))3
⎪⎩ ⎝ dy ⎠ ⎪⎭ ⎩⎪ (f '( x )) ⎭⎪
and  = 
d2x f ''( x ) (f '( x ))3 f ''( x )

dy 2 (f '( x ))3

{1  (f '( x ))2 }3/2


=  …(ii)
f ''( x )

Aakash Educational Services Pvt. Ltd. Regd. Office : Aakash Tower, 8, Pusa Road, New Delhi-110005 Ph.011-47623456
Solutions of Assignment (Set-2) Continuity and Differentiability 203

From (i) and (ii) it follows that

3/2 3/2
2
⎪⎧ ⎛ dx ⎞ ⎪⎫
2
⎪⎧ ⎛ dy ⎞ ⎪⎫
⎨1  ⎜ ⎟ ⎬ ⎨1  ⎜ ⎟ ⎬
⎩⎪ ⎝
dx ⎠ ⎪ ⎪⎩ ⎝ dy ⎠ ⎪⎭
⎭ 
d 2y d2x
dx 2 dy 2

i.e., Statement–1 is true.

dy 1 d 2y d ⎛ dy ⎞
 
dx ⎜⎝ dx ⎟⎠
Statement–2 : We know and 2
dx dx dx
dy

Statement–2 is true and it is a correct explanation of Statement–1.

1  cos 2 x
5. STATEMENT-1 : lim does not exist.
x 0 x

and
STATEMENT-2 : Right hand limit  Left hand limit.
Sol. Answer (1)

1  cos 2 x
Statement–1 : lim
x 0 x

2 | sin x |
lim
x 0 x

We observe that

LHL = – 2

RHL = 2

 LHL  RHL
 Limit does not exist.
Statement–2 : is true.
Statement 1 is true Statement 2 is true, Statement–2 is a correct explanation of Statement–1.

f (x 2 )  f (x)
6. STATEMENT-1 : If f(x) is differentiable increasing function then lim  1 .
x  0 f ( x )  f (0 )

and
STATEMENT-2 : f(x) is an increasing function  f (x) is positive.

Aakash Educational Services Pvt. Ltd. Regd. Office : Aakash Tower, 8, Pusa Road, New Delhi-110005 Ph.011-47623456
204 Continuity and Differentiability Solutions of Assignment (Set-2)

Sol. Answer (2)

f (x2 )  f (x)
Statement–1 : lim
x 0 f ( x )  f (0)

Applying L. Hospital rule

2 xf '( x 2 )  f '( x )
lim = –1
x 0 f '( x )

i.e., Statement–1 is true.


Statement–2 : If f(x) is increasing function then f'(x) > 0
i.e., Statement–2 is true but it is not correct explanation of statement-1.

7. STATEMENT-1 : f ( x )  cos x then  1  f ( x )  1 ⇒ 1  f ( x )  1 .

and

STATEMENT-2 : If f ( x )  [a, b] ⇒ f ( x )  [a, b] .

Sol. Answer (3)

Statement–1 : f ( x )  cos x ⇒  1  cos x  1

f '( x )   sin x ⇒  1  f '( x )  1

Statement–1 is true.

Statement–2 : If f ( x )  [a, b] then it is not necessary

f '( x )  [a, b]

Statement–2 is false.

SECTION - E
Matrix-Match Type Questions
1. For f to be continuous at x = 0 f(0) is given by
Column-I Column-II

ln(1  4 x ) 1
(A) f ( x )  (p)
x 4

ln( 4  x )  ln 4
(B) f ( x )  (q) 0
x

1 1
(C) f ( x )   (r) 4
sin x tan x

1  cos 3 x 3
(D) f ( x )  (s)
x sin 2 x 4

Aakash Educational Services Pvt. Ltd. Regd. Office : Aakash Tower, 8, Pusa Road, New Delhi-110005 Ph.011-47623456
Solutions of Assignment (Set-2) Continuity and Differentiability 205

Sol. Answer A(r), B(p), C(q), D(s)

n(1  4 x )
(A) f (0)  lim =4
x 0 x

n(4  x )  n 4 1
(B) f (0)  lim =
x 0 x 4

⎛ 1 1 ⎞ 1  cos x 1  cos x sin x


(C) f (0)  lim ⎜  ⎟    lim 0
x 0 ⎝ sin x tan x ⎠ sin x 1  cos x x 0 1  cos x

1  cos3 x 1  cos3 x 1  cos3 x


(D) f (0)  lim  lim  lim
x 0 x sin 2 x x 0 sin 2 x x 0 2 x 2
2x 2
2x
Applying L. Hospital rule

3 cos2 x.sin x 3
lim =
x 0 4x 4
2. Match the following :
Column-I Column-II
2 1
(A) lim (sin 2 x ) tan 2x
(p)
x
 2
4

x 1
⎛ 2x  1 ⎞ 
(B) lim ⎜ ⎟ (q) e 2
x ⎝ 2 x  1 ⎠

(C) lim (tan x ) tan 2 x (r) e–1



x
2

⎛ ⎞
(D) lim tan 2 x tan⎜  x ⎟ (s) 1
x
 ⎝ 4 ⎠
4

Sol. Answer A(q), B(r), C(s), D(p)

lim (sin2 x 1) tan2 2 x



tan2 2 x x
(A) lim (sin2 x ) e 4
x
4

⎛ sin2 2 x ⎞
lim ⎜  ⎟
 ⎜ 1 sin2 x ⎟
x ⎝ ⎠
e 4  e 1/2

x 2
⎛ 2x  1 ⎞ lim  .x
(B) lim ⎜  e x  2 x 1  e 1

x  ⎝ 2 x  1 ⎠

Aakash Educational Services Pvt. Ltd. Regd. Office : Aakash Tower, 8, Pusa Road, New Delhi-110005 Ph.011-47623456
206 Continuity and Differentiability Solutions of Assignment (Set-2)

lim tan 2 x logtan x



x
lim (tan x )tan2 x  e 2
(C) 
x
2

logtan x sec 2 x
lim lim 
 cot 2 x  2 tan x cosec 2 2 x
x x
= e 2 e 2

lim  2 sin x.cos x



x
= e 2  e0  1

⎛ ⎞
tan ⎜  x ⎟
⎛ ⎞ ⎝ 4 ⎠
(D) lim tan 2 x.tan ⎜  x ⎟  lim
x
 ⎝ 4 ⎠ x  cot 2 x
4 4

⎛ ⎞
sec 2 ⎜  x ⎟
⎝ 4 ⎠1
Applying L. Hospital rule = lim 2

x  2cosec 2 x
2
4

3. Match the following


Column I Column II

x
(A) If lim 1  x  tan  k , then sin ⎛⎜ 1 ⎞⎟ is (p) 4
x1 2 ⎝k⎠

x k  5k
(B) If lim  500 , then k is (q) 1
x 5 x  5

3 x 1
⎛ 4 ⎞ 3
(C) If lim ⎜ 1  ⎟ is equal to ek, then k is (r) A perfect square
x  ⎝ x  1⎠

d 20
(D)  2cos x.cos3 x   24 k  cos 2 x  220.cos 4 x  , then k is (s) 5
dx 20
(t) An odd number
Sol. Answer A(q, r, t), B(p, r), C(p, r), D(s, t)

1  x  .tan x ⎛⎞
(A) lim  k  lim h tan ⎜ ⎟ (1  h ), 1  x  h, say x  1 ⇒ h  0
x 1 2 h 0 ⎝2⎠
h h 2
 lim cot  k
h 0 2 2 

1
 1. k
/2

2
 k

Aakash Educational Services Pvt. Ltd. Regd. Office : Aakash Tower, 8, Pusa Road, New Delhi-110005 Ph.011-47623456
Solutions of Assignment (Set-2) Continuity and Differentiability 207

1 
 
k 2

⎛ 1⎞
 sin ⎜ ⎟  1 , which is a perfect square and also an odd number.
⎝k ⎠

x k  5k
(B) lim  500
x 5 x 5
 k.5k – 1 = 500
 k = 4, which is a perfect square

⎛ 1 ⎞
1 ⎜ 1 ⎟
3 x 1 x lim 4 ⎝ 3 x ⎠
3 x  1
k ⎛ 4 ⎞ 3 lim 4 1
(C) e  lim ⎜ 1  ⎟  e x  x 1  e x  e4
x  ⎝ x  1⎠

 k = 4 which is a perfect square


(D) 2 cosx cos 3x = cos 4x + cos 2x

d 20
 cos 4 x  cos 2x   420.cos 4 x  220 cos 2x  220  cos 2x  220 cos 4 x 
dx 20
 24k (cos 2x + 220 cos 4x) = 220(cos 2x + 220 cos 4x)
 k = 5 which is an odd number.
4. Match the following
Column I Column II

⎧ 1
⎪ x k cos , x  0
(A) If f  x   ⎨ x is continuous at x = 0, then (p) 2
⎪⎩ 0, x0

the least integral value that k can attain is


(B) If f(x) = (x + 1)cot x is continuous at x = 0 and f(0) = k, (q) A rational number
then [k] is ([.] is G.I.F.)

d4
(C) 4
 x 3 log x   kx then k is (r) 1
dx
(D) If f(x + y) = f(x) f(y)  x, y and f(k) = 2, for some k  R, (s) 6
f (0) = 3, then f (k) is
(t) An odd number
Sol. Answer A(q, r, t), B(p, q), C(q, s), D(q, s)

k 1
(A) lim x cos  0 if k > 1.
x 0 x
Hence the least integral value that k can attain for the function f(x) to be continuous at x = 0 is 1.
1 is rational number. Also 1 is odd

Aakash Educational Services Pvt. Ltd. Regd. Office : Aakash Tower, 8, Pusa Road, New Delhi-110005 Ph.011-47623456
208 Continuity and Differentiability Solutions of Assignment (Set-2)

(B) Since f(x) is continuous at x = 0

lim x cot x ⎛ x ⎞
cot x lim ⎜ ⎟
k  f  0   lim 1  x   e x 0  e x 0⎝ tan x ⎠  e1  e
x 0

Now 2 < e < 3  [k] = [e] = 2 which a rational number.

d4 d3
 4  x 3 log x   3  x 2  3 x 2 .log x 
k
(C)
x dx dx

d3 ⎡ d3 ⎤
= 3
 3 x 2
log x  ⎢ 3  x   0⎥
2

dx ⎣ dx ⎦

d2 ⎡ d
= 3  x 2 log x ⎤⎥
dx 2 ⎢⎣ dx ⎦

d2
= 3  x  2x ln x 
dx 2

d2 
= 6 x ln x 
dx 2

= 6
d
1  ln x   6
dx x

k= 6
(D) f(x + y) = f(x) + f(y)  f(0) = 1

f  x  h  f  x  f h  1
Now f '  x   lim  f  x  lim  f  x  .f '  0   3f  x 
h 0 h h 0 h
 f’(5) = 3f(5) = 3.2 = 6
which is a rational number.
5. Match the function in column I with their domain of definition in column II
Column-I Column-II

⎛x⎞
(A) f ( x )  sin 1 log3 ⎜ ⎟ (p) (–)
⎝3⎠

(B) f ( x )  x 2  5 x  6  2 x  8  x 2 (q) [–2, 0)  (0, 1)

1
(C) f ( x )  (r) [–2, 2]  [3, 4]
2  sin 3 x

1
(D) f ( x )   x2 (s) [1, 9]
log10 (1  x )

Aakash Educational Services Pvt. Ltd. Regd. Office : Aakash Tower, 8, Pusa Road, New Delhi-110005 Ph.011-47623456
Solutions of Assignment (Set-2) Continuity and Differentiability 209

Sol. Answer : A(s), B(r), C(p), D(q)

⎛x⎞
(A) 1  log3 ⎜ ⎟  1
⎝3⎠

x
 3 1   31
3

 1 x  9

(B) x 2  5 x  6  0

 ( x  2)( x  3)  0

x  [ , 2]  [3,  ) …(i)

2x  8  x 2  0

x 2  2x  8  0

( x  4)( x  2)  0

 x  [ 2, 4] …(ii)

By (i) and (ii)

x  [ 2, 2]  [3, 4]

1
(C) f ( x ) 
2  sin3 x

since 2 – sin3x is never equal to zero for any value of x. Hence domain (f) = R.

1
(D) f ( x )   x2
log10 1  x

For f(x) to be real, we have

1  x  1 , 1 – x > 0, i.e., x  0, x < 1 and x  –2

 x  ( , 1)  {0}  [–2, ]

 x  [ 2, 0)  (0, 1)

Aakash Educational Services Pvt. Ltd. Regd. Office : Aakash Tower, 8, Pusa Road, New Delhi-110005 Ph.011-47623456
210 Continuity and Differentiability Solutions of Assignment (Set-2)

6. Match column-I to column-II according to the given conditions.


Column-I Column-II

e tan x
– e sin x 
(A) xlim equals (p) 1
0 (tan x – sin x )

∫ xe dx
x

0
1
(B) lim equals (q)
x 0 x sin x 2

x 1 1
(C) lim equals (r) –
x  –
4x 2  x  2 2

3
(D) lim (sin x )2 n ,where n  N, is not equal to (s)
n 4
(t) 2
Sol. Answer : A(p), B(q), C(r), D(q, r, s, t)

lim
e tan x – e sin x
lim

e sin x e tan x – sin x – 1 
 e0  1  1
 
(A) x  0 tan x – sin x = x  0
 tan x – sin x 
x

∫ xe
x

0
(B) lim
x0 x sin x

xe x
= lim (Using L – H Rule)
x0 x cos x  sin x

xe x  e x 1
= lim  (Using L – H Rule)
x0 – x sin x  cos x  cos x 2

x 1 x 1 1
(C) xlim = xlim –
 –
1 2  –
1 2 2
|x| 4  –x 4  
x x2 x x2

 sin x 
2n
(D) nlim


Case (i) if sinx ¹ (±1)

lim  sin x   0
2n
 n

Case (ii) If sinx = ±1

lim  1  1
2n
 n 

Aakash Educational Services Pvt. Ltd. Regd. Office : Aakash Tower, 8, Pusa Road, New Delhi-110005 Ph.011-47623456
Solutions of Assignment (Set-2) Continuity and Differentiability 211

7. Match column-I to column-II according to the right hand limit and left hand limit at the given point, where [ ]
represents the greatest integer function. (where [ ] is G.I.F.)
Column-I Column-II

⎡ sin  x  ⎤
(A) lim ⎢ ⎥ (p) 0
⎣⎢  x  ⎦⎥
x0

⎡ tan[ x ] ⎤
(B) lim ⎢ ⎥ (q) 10
x0
⎣ [x] ⎦

(C) lim  x  (r) 9


x  10

(D) lim cos x  (s) –1


x
2

e cos x – 1
(t) lim
 cos x
x
2

Sol. Answer : A(p), B(t), C(q, r), D(p, s)

⎡ sin 0  h  ⎤ ⎡ 0 ⎤
(A) R H L = hlim ⎢ ⎥  ⎢ ⎥ , not defined
0
⎢⎣ 0  h  ⎥⎦ ⎣ 0 ⎦

⎡ sin 0 – h  ⎤
L H L = hlim ⎢ ⎥   sin 1  0
⎣⎢ 0 – h  ⎦⎥
0

⎡ tan 0  h  ⎤ 0
(B) R H L = hlim ⎢ ⎥
⎣⎢ 0  h  ⎦⎥ 0
0 not defined

⎡ tan 0 – h  ⎤
L H L = hlim ⎢ ⎥ 1
⎣⎢ 0 – h  ⎦⎥
0

(C) R H L = hlim
0
10  h   10

L H L = hlim
0
10  h   9

⎡ ⎛ ⎞⎤
(D) R H L = hlim cos ⎜  h ⎟ ⎥  –1
0 ⎢ 2
⎣ ⎝ ⎠⎦

⎡ ⎛ ⎞⎤
L H L = hlim cos ⎜ – h ⎟ ⎥  0
0 ⎢ 2
⎣ ⎝ ⎠⎦

Aakash Educational Services Pvt. Ltd. Regd. Office : Aakash Tower, 8, Pusa Road, New Delhi-110005 Ph.011-47623456
212 Continuity and Differentiability Solutions of Assignment (Set-2)

8. Match the following :


Column-I Column-II

16 xy
(A) 4x2 + 9y2 = 36 (p) y II  
( y  2 x )3
2

2(3y 4  8y 2  5)
(B) x2 + y2 = 4 (q) y III  
y8

12 x
(C) y = tan(x + y) (r) y III  
y5

16
(D) x3 + y3 – 6xy = 0 (s) y II  
9y 3

Sol. Answer A(s), B(r), C(q), D(p)

(A) 4 x 2  9 y 2  36

 4 x  9 yy '  0

4x
 y'  
9y

 4  9 yy   9( y ')2  0

16 x 2
 9 yy   4 
9y 2

4(4 x 2  9 y 2 )
 9 yy   
9y 2

16
 y '' 
9y 3

(B) x 2  y 2  4

 x  yy   0

 1  yy '' ( y ')2  0

x2 x2  y 2
 yy   1  
y2 y2

4
 y   
y3
12 12x
 y   y'  y   
4
y y5

Aakash Educational Services Pvt. Ltd. Regd. Office : Aakash Tower, 8, Pusa Road, New Delhi-110005 Ph.011-47623456
Solutions of Assignment (Set-2) Continuity and Differentiability 213

(C) y = tan (x + y)
 y' =sec2 (x + y) (1 + y')

1 y 2
 y'  
y2

2 2(1  y 2 )
 y ''   .y '  
3
y y5

2y 6  (1  y 2 )5 y 4
 y   2 .y '
y 10

2(3 y 4  8 y 2  5)
 y   
y8

(D) x 3  y 3  6 xy  0

 x 2  y 2 y ' 2( xy ' y )  0

 x 2  y '( y 2  2 x )  2y  0

 2 x  2y ( y ')2  ( y 2  2 x )y '' 4 y '  0

4 y ' 2y ( y ')2  2 x 16 xy
 y ''   y ''  
2
( y  2x ) ( y  2 x )3
2

9. Match the following :


Column-I Column-II

1
(A) f ( x )  (p) lim f ( x )  0
x 2 x 0

x  sin x
(B) f ( x )  (q) f is continuous on R
x  sin x


(C) f ( x )  x sin , f (0)  0 (r) lim f ( x )  0
x x 

1
(D) f ( x )  tan 1 (s) lim f ( x ) does not exist
x x0

Sol. Answer A(r, s), B(p), C(p, q), D(r, s)

1
(A) f ( x ) 
x 2

1
 lim 0
x  x 2

Aakash Educational Services Pvt. Ltd. Regd. Office : Aakash Tower, 8, Pusa Road, New Delhi-110005 Ph.011-47623456
214 Continuity and Differentiability Solutions of Assignment (Set-2)

x  sin x
(B) lim
x 0 x  sin x

sin x
1
 lim x 0
x 0 sin x
1
x

(C) f ( x )  x sin
x
At x = 0


LHL = lim x sin 0
x 0 x


RHL = lim x sin 0
x 0 x
f(0) = 0
i.e., f is continuous on R.

1 1
(D) f(x) = tan
x
At x = 0

1 
RHL = lim tan1 
x 0 x 2

1 
LHL = lim tan1 
x 0 x 2
LHL  RHL
i.e., lim f ( x ) does not exist.
x 0

10. Match the limit in column I with their values in column II


Column-I Column-II

cos x  cos 3 x
(A) lim (p) 3
x 0 x(sin x  sin 3 x )

sin 4 x
(B) lim (q) –2
x 0 tan 2 x

x 4  2x 3  3 1
(C) lim (r)
x  4
2x  x  2 2

3x 2  3x  9  3x  3
(D) lim (s) 2
x 2 x 2

Aakash Educational Services Pvt. Ltd. Regd. Office : Aakash Tower, 8, Pusa Road, New Delhi-110005 Ph.011-47623456
Solutions of Assignment (Set-2) Continuity and Differentiability 215

Sol. Answer : A(q), B(s), C(r), D(s)

cos x  cos3 x
(A) lim
x 0 x (sin x  sin3 x )

⎛ 3x  x ⎞ ⎛ 3x  x ⎞
2 sin ⎜ ⎟ sin ⎜ ⎟
⎝ 2 ⎠ ⎝ 2 ⎠ 2 sin 2 x.sin x
lim = lim  2
x 0 ⎛ x  3x ⎞ ⎛ x  3x ⎞ x 0  x.2cos 2 x.sin x
x.2cos ⎜ ⎟ .sin ⎜ 2 ⎟
⎝ 2 ⎠ ⎝ ⎠

⎛ sin 4 x ⎞
sin 4 x ⎜ 4 x ⎟ (4 x )
(B) lim  lim ⎝ ⎠ 2
x 0 tan 2 x x 0 ⎛ tan 2 x ⎞
⎜ 2x ⎟ (2 x )
⎝ ⎠

2 3
4 3  1
x  2x  3 x x4 1 0  0 1
lim  lim  
(C) x  2 x 4  x  2 x  1 2 200 2
2 
x3 x 4

3x2  3x  9  3x  3
(D) lim
x 2 x 2

(3 x 2  3 x  9)  (3 x  3)
= lim
x 2 ( x  2)( 3 x 2  3 x  9  3 x  3)

(3 x 2  12) 3( x  2)( x  2) 3  4
= lim = lim  2
x 2 ( x  2)(3  3) x 2 ( x  2)(6) 6

SECTION - F
Integer Answer Type Questions

1. The number of points where f  x   max  sin x , cos x  , x   2, 2  is not differentiable is______.

Sol. Answer (8)

y f (x) = max {| sinx |, | cosx |}

x
0  3 5 7  2
4 4 4 4

The function is even. There are four points in (0, 2) where it is not differentiable. Hence in [–2, 2] there are
8 such points

Aakash Educational Services Pvt. Ltd. Regd. Office : Aakash Tower, 8, Pusa Road, New Delhi-110005 Ph.011-47623456
216 Continuity and Differentiability Solutions of Assignment (Set-2)

2. The number of points where g  x   min  x  3 , 9  x 2   x  R  is not differentiable is _______.

Sol. Answer (3)

2
9–x

y = ||x| = – 3|

It is clear from the graph that there are three points where there is sharp corner. Hence there are three points
where function is not differentiable.
3. If the right hand derivative of h(x) = {x} ({.} is fractional part of x) exists at x = 1 and it is equal to_______.
Sol. Answer (1)

1  h  1 h0
lim  lim  lim 1  1
h 0 h h 0 h h 0

4. If y = f(x) and y4 – 4y + x = 0. If f(–8) = 2 then the value of |28 f(–8)| is _____.


Sol. Answer (1)
y4 – 4y + x = 0
at y = 2, x – 8
Differentiating the equation


4 y3 – 1  dy
dx
1 0

dy –1


dx 4 y 3 – 1 
1
 f  x   –

4 y3 – 1 
1 1
f   –8   – –

4 2 –1 3
 28

 |28 f'(–8)| = 1

⎛ 1⎞ ⎛ 1⎞ ⎛3⎞
5. If f(x) = f(1 – x) and f(x) is differentiable at every real value of x then the value of f  ⎜ ⎟  f  ⎜ ⎟  f  ⎜ ⎟ is ____.
2
⎝ ⎠ 4
⎝ ⎠ ⎝4⎠
Sol. Answer (0)
f(x) = f(1 – x)
 f'(x) = –f'(1 – x)
1
at x =
2

Aakash Educational Services Pvt. Ltd. Regd. Office : Aakash Tower, 8, Pusa Road, New Delhi-110005 Ph.011-47623456
Solutions of Assignment (Set-2) Continuity and Differentiability 217

⎛ 1⎞ ⎛ 1⎞
f  ⎜ ⎟  –f  ⎜ 1– ⎟
2
⎝ ⎠ ⎝ 2 ⎠

⎛ 1⎞ ⎛ 1⎞
f ⎜ ⎟  f ⎜ ⎟  0
2
⎝ ⎠ ⎝2⎠

⎛ 1⎞
 f ⎜ ⎟  0 …(i)
⎝2⎠
1
Also x =
4
⎛ 1⎞ ⎛3⎞
f  ⎜ ⎟  –f  ⎜ ⎟
⎝4⎠ ⎝4⎠

⎛ 1⎞ ⎛3⎞
 f ⎜ ⎟  f ⎜ ⎟  0 …(ii)
⎝4⎠ ⎝4⎠
By (i) and (ii)

⎛ 1⎞ ⎛ 1⎞ ⎛3⎞
f ⎜ ⎟  f ⎜ ⎟  f ⎜ ⎟  0
⎝2⎠ ⎝4⎠ ⎝4⎠

2k
 
cot x
6. If the value of lim 1  sin x  sin2 x is k then is equal to ____.
x0 e
Sol. Answer (2)
 sin x  sin x .cot x
 
cot x lim 2

lim 1  sin x  sin2 x  e x0


x0

⎛ sin x  sin2 x ⎞
lim ⎜ ⎟⎟ x  x2
x0⎜ tan x lim
= e ⎝ ⎠
e x0 x
 e1  e

2k 2  e
k = e   2
e e

7. The value of lim


e sin
3
x
– cos  sin x   3
 is _____.
3
x0 x
Sol. Answer (1)

lim
e sin
3
x
– cos  sin x   3

3
x0 x

 
3
e x – cos x 3 sin x
lim (using lim  1)
x0 x 3 x0 x

lim
e  3 x    sin( x )3x 
x3 2 3 2

x0 (3 x 2 )

= lim
x0
 3
e x  (sin x 3 )  e 0  0  1 
Aakash Educational Services Pvt. Ltd. Regd. Office : Aakash Tower, 8, Pusa Road, New Delhi-110005 Ph.011-47623456
218 Continuity and Differentiability Solutions of Assignment (Set-2)

1⎡ 6 ⎤
8. Let f(x) is a function satisfying the condition f ( x )  f ( x  6)  and f(x) > 0  x  R then if
3 ⎢⎣ f ( x  7) ⎥⎦

lim f ( x )  m then the value of m is _____.


x 

Sol. Answer (3)

∵ lim f ( x )  lim f ( x  6)  lim f ( x  7)  m


x  x  x 

then from the given condition

1⎡ 6⎤
m ⎢ m ⎥
3⎣ m⎦
 3m2 = m2 + 6
 2m2 = 6

 m 3

so, lim f ( x )  3
x 

so the value of m = 3

1 2mx 3
9. If lim (e  ex  x)  then the value of m is _____.
x 0 x2 2
Sol. Answer (1)

1 3
Given that lim 2
(e 2mx  e x  x ) 
x 0 x 2

⎛ 1⎞
x (2m  2)  x 2 ⎜ 2m 2  ⎟  x 3 (..)  ...
⎝ 3⎠ 3
 lim 2
 m=1
x 0 x 2

⎡ m sin x ⎤
10. If lim ⎢  8 then the value of m is ([.] in the greatest integer function).
x 0 ⎣ x ⎥⎦

Sol. Answer (9)


We know that

sin x
1
x

m sin x ⎡ m sin x ⎤
 m  xlim
0 ⎢ ⎥  (m  1)
x ⎣ x ⎦

⎡ m sin x ⎤
But given that lim ⎢ 8
x 0 ⎣ x ⎥⎦
so m – 1 = 8 m=9

Aakash Educational Services Pvt. Ltd. Regd. Office : Aakash Tower, 8, Pusa Road, New Delhi-110005 Ph.011-47623456
Solutions of Assignment (Set-2) Continuity and Differentiability 219

11. Let f : »  » and g : »  » be respectively given by f(x) = |x| + 1 and g(x) = x2 + 1. Define h : »  »
by
⎧max {f ( x ), g ( x )} if x  0,
h( x )  ⎨
⎩ min {f ( x ), g ( x )} if x  0.

The number of points at which h(x) is not differentiable is [JEE(Advanced)-2014]


Sol. Answer (3)

1
12. Let f : »  » be a continuous odd function, which vanishes exactly at one point and f (1)  . Suppose that
2
x x
F(x) 1
F(x)  ∫ f (t ) dt for all x  [–1, 2] and G( x )  ∫ t | f (f (t )) | dt for all x[ 1, 2] . If lim
x 1 G( x )

14
, then the value
1 1

⎛ 1⎞
of f ⎜ ⎟ is [JEE(Advanced)-2015]
⎝2⎠
Sol. Answer (7)
1 1
f (1)  ; f (  x )  f ( x ); f ( 1)   and f(x) is zero only at one point
2 2
x
F(x)  ∫ f (t ) dt x [ 1, 2]
1

F ( x )  f ( x )
x
G( x )  ∫ f | f (f (t )) | dt
1

 G( x )  x | f (f ( x )) |

F(x) F ( x ) f (x)
lim  lim  lim
x 1 G( x ) x 1 G( x ) x 1 x | f (f ( x )) |

1
2  1
=
⎛ 1 ⎞ 14
f⎜ ⎟
⎝2⎠

⎛ 1⎞
 f⎜ ⎟ 7
⎝2⎠

⎛ 1⎞ ⎛ 1⎞
 f ⎜ ⎟  7 as f ⎜ ⎟ can not be negative
⎝2⎠ ⎝2⎠
Aakash Educational Services Pvt. Ltd. Regd. Office : Aakash Tower, 8, Pusa Road, New Delhi-110005 Ph.011-47623456
220 Continuity and Differentiability Solutions of Assignment (Set-2)

SECTION - G
Multiple True-False Type Questions
1. STATEMENT-1 : y = [x] ([.] denotes greatest integer function) is not a continuous function.
STATEMENT-2 : y = {x} ({.} denotes fractional function) is discontinuous at integral points.
STATEMENT-3 : y = 7x is continuous in its domain.
(1) T F T (2) TTT (3) FFF (4) FFT
Sol. Answer (2)

y = [x]

STATEMENT – 1 : True from graph

y = {x }
STATEMENT – 2 : True from graph

STATEMENT – 3 : True from graph

sin x
2. STATEMENT-1 : lim exists.
x 0 x
STATEMENT-2 : |x| is differentiable at x = 0.

tan kx
STATEMENT-3 : If lim  3 , then k = 15.
x 0 sin 5 x
(1) T F T (2) TTT (3) F F F (4) FFT
Sol. Answer (1)

sin x
STATEMENT – 1 : True. As, lim 1
x 0 x
STATEMENT – 2 : False. The graph of |x| has sharp corner at x = 0

⎛ tan kx ⎞
⎜ ⎟
⎝ kx ⎠ . kx  3
STATEMENT – 3 : True lim
x  0 ⎛ sin5 x ⎞ 5 x
⎜ ⎟
⎝ 5x ⎠

k
1.  3 ⇒ k  15
5

Aakash Educational Services Pvt. Ltd. Regd. Office : Aakash Tower, 8, Pusa Road, New Delhi-110005 Ph.011-47623456
Solutions of Assignment (Set-2) Continuity and Differentiability 221

log  x 
STATEMENT-1 : xlim  0 , where [x] represents the integral part of x.
3. 
x

sec 2 x – 1
STATEMENT-2 : lim does not exist.
x0 x

1
STATEMENT-3 : lim
x2
 x – 1 x – 2 = 1
(1) T T T (2) TTF (3) FTF (4) FFF
Sol. Answer (2)

log[ x ]
Statement –1 : xlim 0
 [x]

ln x
As is decreasing for x > 1
x

ln x ln( x ) ln( x  1)
  
x (x) x 1

Using Sandwich theorem

ln[ x ]
lim 0
x  [x]

[tan x ]
Statement – 2 : lim
x 0 x

[tan( h )]
LHL = lim  1
x 0 h h

[tan h]
RHL = lim 1
x 0 h h

1 1
lim ( x  2)
Statement – 3 : lim (1  x  2) x  2  e x 2 x 2  e1
x 2

A sin x  B log(1  x 2 )  C (1– cos x )


4. lim  2, then
x0 x2
STATEMENT-1 : A = 1
STATEMENT-2 : 2B + C = 2
STATEMENT-3 : A + 2B + C = 5
(1) T T T (2) TTF (3) FTF (4) FFF

Aakash Educational Services Pvt. Ltd. Regd. Office : Aakash Tower, 8, Pusa Road, New Delhi-110005 Ph.011-47623456
222 Continuity and Differentiability Solutions of Assignment (Set-2)

Sol. Answer (4)

⎛ x3 x5 ⎞ ⎛ x4 ⎞ ⎛ ⎛ x2 x4 ⎞ ⎞
A⎜ x –   .... ⎟  B ⎜ x 2 –  .... ⎟  C ⎜ 1– ⎜ 1–  – ⎟ ⎟  ......
⎝ 3! 5! ⎠ ⎝ 2 ⎠ ⎝ ⎝ 2! 4! ⎠ ⎠
lim 2
x0 x2

⎛ C⎞ ⎛ A⎞
x( A)  x 2 ⎜ B  ⎟  x 3 ⎜ – ⎟  ......
⎝ 2 ⎠ ⎝ 3! ⎠
lim 2
2
x0 x

C
 A = 0, B + =2
2
Hence all the statements are false.

SECTION - H
Aakash Challengers Questions
1. ABC is an isosceles triangle inscribed in a circle of radius r. If AB = AC and h is the altitude from A to BC. If the


ABC has perimeter P and area  then lim 512r equals
h 0 p3

Sol. We have BC = 2BD, AD = h

and OD = h – r

 BC  2 r 2  (h  r )2 = 2 2hr  h2

 AB  2hr

The perimeter of the ABC


A

 P  2 AB  BC  2 ⎡ 2hr  h 2  2hr ⎤
⎣⎢ ⎦⎥ r

r O
2
Area of ABC = h 2hr  h B C
D

 h 2hr  h2
Now 
P3 3
8 ⎛⎜ 2hr  h 2  2hr ⎞⎟
⎝ ⎠

 h 2hr  h 2
 lim 512  lim 512r . 4
h 0 p3 h 0 3
8 ⎛⎜ 2hr  h 2  2hr ⎞⎟
⎝ ⎠

Aakash Educational Services Pvt. Ltd. Regd. Office : Aakash Tower, 8, Pusa Road, New Delhi-110005 Ph.011-47623456
Solutions of Assignment (Set-2) Continuity and Differentiability 223

2. Let f(x) = ||x| – 1|, g(x) = |x| + |x – 2|, h(x) = max. {1, x, x3}. If a, b, c are the no. of points where f(x), g(x)
and h(x) are not differentiable, then the value of a + b + c is ……
Sol. Answer (6)

(–10) (10)

Graph of f (x)  | x |  1

g( x ) | x |  | x  2 |

(0, 0)

(20)

h( x )  max.{1, x, x 3 }

 6 points.

a1 a2 x a3 x 2 an x n 1 dy
3. Let y  1     ....  . Find .
x  a1 ( x  a1 )( x  a2 ) ( x  a1 )( x  a2 )( x  a3 ) ( x  a1 )( x  a2 )( x  a3 )...( x  an ) dx

⎛ a1 ⎞ a2 x
Sol. y  ⎜ 1  x  a ⎟  ( x  a )( x  a )  ....
⎝ 1⎠ 1 2

⎛ x a2 x ⎞
y ⎜  ⎟  ...
⎝ x  a1 ( x  a1 )( x  a2 ) ⎠

x ⎛ a2 ⎞ a3 x 2
y ⎜1  ⎟  ...
x  a1 ⎝ x  a2 ⎠ ( x  a1 )( x  a2 )( x  a3 )

Aakash Educational Services Pvt. Ltd. Regd. Office : Aakash Tower, 8, Pusa Road, New Delhi-110005 Ph.011-47623456
224 Continuity and Differentiability Solutions of Assignment (Set-2)

x2 a3 x 2
y   ...
( x  a1 )( x  a2 ) ( x  a1 )( x  a2 )( x  a3 )

x3
y  ... and so on.
( x  a1 )( x  a2 )( x  a3 )

Hence y can be sum use as

xn
y
( x  a1 )( x  a2 )...( x  an )

log y  n log x  log( x  a1 )  log( x  a2 )  log( x  a3 )  ...

1 dy n 1 1 ⎛1 1 ⎞ ⎛1 1 ⎞
   .... = ⎜  ⎟⎜  ⎟  ....
y dx x x  a1 x  a2 ⎝ x x  a1 ⎠ ⎝ x x  a2 ⎠

a1 a2
= x( x  a )  x( x  a )  ...
1 2

dy y ⎡ a1 a2 ⎤
  ⎢   ...⎥
dx x ⎣ x  a1 x  a2 ⎦

⎧ cosec x 
⎪ (sin x  cos x ) , 
2
x0

If lim f ( x ) exists and is equal to p, then find p and q where, f ( x )  ⎪⎨
1 2 3
4.
x0 ex ex ex 
⎪ , 0x
1 3 2
⎪ 2  1
⎪⎩ pe x  qe x

⎧ cosec x 
⎪(sin x  cos x ) ; 
2
x0

⎪ 1 2 3
Sol. f ( x )  ⎨
e x  ex  ex 
⎪ ; 0x
⎪ 2
1
1
3 2
⎪⎩ pe x  qe x

1 2 3 1 2
eh  eh  eh 1 e h  e h
f (0 )  lim  lim
h 0 1 3 h 0 2
2  1 1
2
pe h  qe h pe  qe h

2 1
 
4e h e e 1 1 e
= lim  
h 0 2 1 q
 qe
pe 2e h  qe 1

Aakash Educational Services Pvt. Ltd. Regd. Office : Aakash Tower, 8, Pusa Road, New Delhi-110005 Ph.011-47623456
Solutions of Assignment (Set-2) Continuity and Differentiability 225

f(0–) = lim (sin(0  h )  cos(0  h )cosec(0 h )


h 0

= lim (cos h  sin h )cosec h


h 0

1
lim (cos h sin h 1)
(  sin h )
= e h 0

1 sin h cos h
lim
sin h
= e h 0

cos h  sin h
lim
cos h
= e h 0 e

Put f (0 )  f (0 )

e
 e
q

 q = 1 and p  R

k |x|  1 | x | ln k
5. Evaluate lim , k  0.
x 0 x2

| x|
Sol. lim k  1 | x | ln k , here we have to find RHL and LHL.
x 0 x2

k |x|  1  | x | ln k k |h|  1 | h | ln k
RHL = lim = lim
x 0 x2 h 0 h2

k h  1  h ln k k h log k  log k
= lim 2 = lim
h0 h h 0 2h

k h (log k )2 1
= lim  (log k )2
h 0 2 2

k |0h|  1 | 0  h | ln k 1
LHL = lim  (log k )2
h 0 2 2
h
6. Let P(x) be a polynomial of degree 4, with P(2) = –1, P I(2) = 0, P II(2) = 2, P III(2) = –12 and P IV (2) = 24.
Calculate P II(1).....
Sol. Let p(x) = ax4 + bx3 + cx2 + dx + e
 p'(x) = 4ax3 + 3bx2 + 2cx + d
 p''(x) = 12ax2 + 6bx + 2c
 p'''(x) = 24ax + 6b

Aakash Educational Services Pvt. Ltd. Regd. Office : Aakash Tower, 8, Pusa Road, New Delhi-110005 Ph.011-47623456
226 Continuity and Differentiability Solutions of Assignment (Set-2)

 piv(x) = 24a

 piv(2) = 24a

 24 = 24a a=1

Now p'''(2) = 48a + 6b

 –12 = 48 + 6b

 6b = –60  b = –10

Also p''(2) = 12a(2)2 + 12b + 2c

 2 = 48 – 120 + 2c

 –46 + 120 = 2c  c = 37

Hence p''(1) = 12a + 6b + 2c = 12 – 60 + 74 = 26

x3
7. If lim  1 , a R+, then the value of a + b + 1975 is .........
x 0 a  x (bx  sin x )

x3
Sol. Given, Lt
1
1
x 0
⎛ x⎞2
a ⎜ 1  ⎟  (bx  sin x )
⎝ a ⎠

x3
 Lt
x 0 ⎧ 1⎛ 1⎞ ⎫
⎪⎪ 1 ⎛ x ⎞ 2 ⎜  2 ⎟ ⎛ x ⎞2 ⎪⎪ ⎪⎧ x3 ⎪⎫
a ⎨1  ⎜ ⎟  ⎝ ⎠
⎜ ⎟  .....⎬ ⎨ x(b  1)   ....⎬
⎪ 2⎝a ⎠ 2 ⎝a⎠ ⎪ ⎩⎪ 3! ⎭⎪
⎩⎪ ⎭⎪

3!
  1 and b = 1
a
 a = 36, b = 1
 a + b + 1975 = 36 + 1 + 1975 = 2012

n
⎛ 1 n 4 ⎞
8. Evaluate; nLt ⎜
 ⎜
⎟⎟ .
⎝ 2 ⎠

1
n
⎛ 1 n 4 ⎞ 4
n 1 lim
1 4 n 1
.
1
log 4
Lt ⎜ = e log 4 =
⎜ 2 ⎟⎟ = Lt = e2 4 =2
Sol. n .n = e n  2 1/ n
⎝ ⎠ e n 
2

x2 1
9. If the normal of f(x) = 0 at x = 0 given by 3x – y + 3 = 0 such that lim  , then
x 0 f ( x 2 )  5f (4 x 2 )  4f (7 x 2 ) 
the value of 7 – 177 is ……

Aakash Educational Services Pvt. Ltd. Regd. Office : Aakash Tower, 8, Pusa Road, New Delhi-110005 Ph.011-47623456
Solutions of Assignment (Set-2) Continuity and Differentiability 227

Sol. Given, y = 3x + 3

dy
 3
dx

⎛ dy ⎞
 ⎜ dx ⎟ 3
⎝ ⎠ x 0

1
 Slope of normal =  at x = 0
3

1 x2
   lim
 x 0 f ( x 2 )  5f (4 x 2 )  4f (7 x 2 )

2x
= xlim
0 2 xf ( x )  40 xf (4 x 2 )  56 xf (7 x 2 )
2

1
= xlim
0 f ( x )  20f (4 x 2 )  28f (7 x 2 )
2

1
=
1 1 1
  20   28 
3 3 3

1 3 1
=  
20  29 9 3
3
 =3
 7 – 177 = 37 – 177 = 2187 – 177 = 2010
10. Let f(x) = [sinx + cosx], [.] = greatest integer function,  x  [0, 2]

1
g( x ) 
log | x |

1
h( x ) 
| x | 2 .

If a, b and c are the number of points of discontinuities of f(x), g(x) and h(x) respectively then the value of a
+ b + c is ……
Sol. f(x) = [sinx + cosx]

⎧  3 3 7 ⎫
 a = 6, where x  ⎨ , , , , , 2 ⎬
⎩ 2 4 2 4 ⎭

1
 g( x ) 
log | x |

Aakash Educational Services Pvt. Ltd. Regd. Office : Aakash Tower, 8, Pusa Road, New Delhi-110005 Ph.011-47623456
228 Continuity and Differentiability Solutions of Assignment (Set-2)

 b = 3, where x = {–1, 0, 1}

1
 h( x ) 
| x | 2

 c = 2, where x = {–2, 2}
 a + b + c = 6 + 3 + 2 = 11

n 1
⎡ r⎤
ar ∑
11. Let an  ∑
⎢ x  n ⎥, n  N, where [·] = greatest integer function and bn 
r 0 ⎣ ⎦
r 1
n2
. If lim bn  f ( x ), then the
n 

value of lim f ( x ) is …
x 1

⎡ 1⎤ ⎡ 2⎤ ⎡ n  1⎤
Sol.  an  [ x ]  ⎢ x  ⎥  ⎢ x  ⎥    ⎢ x   [nx ]
⎣ n⎦ ⎣ n⎦ ⎣ n ⎥⎦

∑a
r 1
r
a1  a2  a3    an [ x ]  [2 x ]  [3 x ]    [nx ]
 bn  2
 =
n n2 n2

[ x ]  [2 x ]    [nx ] x
Now, lim bn  lim 
2 2
n  n  n

x
 f (x) 
2

1
 lim f ( x ) 
x 1 2

12. Let f(x) = [n + p sinx],  x [0, ], n N and p is a prime number
g(x) = [sinx + cosx],  x [0, ]

h(x) = [ 2 sinx],  x [–2, 2], where [.] = Greatest integer function

If a, b, c are number of points, where f(x), g(x) and h(x) are not differentiable respectively and a + b + c = 35,
then the value of p.
Sol. Given, f(x) = [n + p sinx]
 f(x) is not differentiable at points,
where n + p sinx = an integer = k(say)

k n
 sinx =
p

Aakash Educational Services Pvt. Ltd. Regd. Office : Aakash Tower, 8, Pusa Road, New Delhi-110005 Ph.011-47623456
Solutions of Assignment (Set-2) Continuity and Differentiability 229

In [0, ], 0  sinx  1


 0  p sinx  p
 n  n + p sinx  n + p
 n + p sinx = n, n + 1, n + 2, ……, n + p
 p sinx = 0, 1, 2, ……, p

1 2 p 1
 sin x  0, , ,, ,1
p p p

Now, sinx = 0  x = 0, 

1 ⎛ 1⎞ ⎛ 1⎞
sin x  ⇒ x  sin1 ⎜ ⎟ ,   sin1 ⎜ ⎟
p p
⎝ ⎠ ⎝p⎠

………………………
………………………

p 1 ⎛ p  1⎞ 1 p  1
sin x  ⇒ x  sin1 ⎜ ⎟ ,   sin
p ⎝ p ⎠ p


sin x  1 ⇒ x 
2

Thus, a = 2p + 1

⎡ ⎛  ⎞⎤
Also, g(x) = [sinx + cosx] = ⎢ 2 sin ⎜ x  ⎟ ⎥
⎣ ⎝ 4 ⎠⎦

⎛ ⎞
 2 sin ⎜ x  ⎟  1, 0, 1
⎝ 4 ⎠

 1 1
 sin ⎛⎜ x  ⎞⎟   , 0,
⎝ 4 ⎠ 2 2

  3 5
 x  , , ,
4 4 4 4

3 
 x , 0, , 
4 2

 b=4

Also, h(x) = [ 2 sin x ]

Aakash Educational Services Pvt. Ltd. Regd. Office : Aakash Tower, 8, Pusa Road, New Delhi-110005 Ph.011-47623456
230 Continuity and Differentiability Solutions of Assignment (Set-2)

 2 sin x  1, 0, 1

1 1
 sin x   , 0,
2 2

5 7 3   3 5 7


 x , , , , 0, , 2,  ,  2, , , ,
4 4 4 4 4 4 4 4
 c = 13
Now, a + b + c = 40
 2p + 1 + 4 + 13 = 40
 2p + 1 = 40 – 17 = 23
 2p = 22
 p = 11

  

Aakash Educational Services Pvt. Ltd. Regd. Office : Aakash Tower, 8, Pusa Road, New Delhi-110005 Ph.011-47623456

You might also like